Download as doc, pdf, or txt
Download as doc, pdf, or txt
You are on page 1of 61

Study Guide Infection and Infectious Diseases

STUDY GUIDE
INFECTION &
INFECTIOUS DISEASES

PROGRAM STUDI PENDIDIKAN DOKTER


FAKULTAS KEDOKTERAN
UNIVERSITAS UDAYANA

Udayana University Faculty of Medicine, DME, 2017 1


Study Guide Infection and Infectious Diseases

CONTENTS

Table of contents ... 1


Introduction . 2
Curriculum 4
The Seven General Core Competencies. 6
Block Team ................... 7
Facilitators 8
Time Table Regular Class 9
Time Table English Class .. 9
Students Project.. 14
Assessment method ... 16
Learning Program .. 17
Lecture . 17
Learning Task and Self Assessment.17
Curriculum Mapping ... 60
Reference 61

Udayana University Faculty of Medicine, DME, 2017 2


Study Guide Infection and Infectious Diseases

INTRODUCTION

Due to the application of integrated curriculum at the Faculty of Medicine


Udayana University, the discipline-based subjects of the previous curriculum such
as Biology, Anatomy, Physiology, Internal Medicine, etc have been integrated and
incorporated into several blocks. One of these blocks is Infections and Infectious
Diseases. In this block will be explained in general about pathogenesis,
pathophysiology, sign, symptoms, clinical features, diagnosis, and management of
certain infectious diseases commonly occur in community.
This book aims to give general information for medical students about infections
and infectious diseases and important for facilitators and resource person while
facilitating or guiding the students in learning process. This study guide consists of
general information on learning time table, block team members, facilitators, and the
core curriculum including learning outcomes, learning situations, learning tasks and
self-evaluation items.
The block Infection and Infectious Diseases has the equivalent of 3 (three)
credits. As a block of six credits, the learning processes will be carried out for 38
days starts from October, 6th 2017 as shown in the Time Table. Evaluation for
infection and infectious disease will be held on 9th November 2017. Final mark of
this block is combination between result of basic and clinical examination. During
the 38 days of learning activities, the students will discuss several topics in varied
forms of learning situations such as independent learning, small group discussion,
lecture, and skill lab.
More than half of the learning material must be learned independently and in
small group discussions. A lecture is given only to emphasize crucial things or
objectivesof material and to prepare the students before discussion. For small group
discussion, the students will be given learning tasks to solve and discuss. After
discussion, students also have to evaluate their learning progress independently
(self assessment).
From this block, we hope every medical student have knowledge and skill to
diagnose and manage infections and certain infectious diseases commonly occur in
community, as a frontline in community health.
Since the integrated curriculum of the Faculty of Medicine Udayana University is
still in progress, this Study Guide will also, naturally, have some revisions in the
future. Therefore, we kindly invite readers to give any comments or suggestions for
its improvement and development.

Planners

Udayana University Faculty of Medicine, DME, 2017 3


Study Guide Infection and Infectious Diseases

CURRICULUM OF THE BLOCK

AIMS
To comprehend the biology of the infectious diseases
To apply and interpret common laboratory diagnosis of infectious diseases
To diagnose and manage common infectious diseases
To carry out basic immunization in children

LEARNING OUTCOMES
Comprehend the practical and clinical implications of the biology of infection
Apply the general principles of approach to patients with infectious diseases
Apply and interpret common laboratory diagnosis of common infectious
diseases
Apply the basic principles of immunization in children
Diagnose and manage common bacterial infections (common Gram positive
and negative, spirochetal)
Diagnose and manage common parasitic infections (common nematode,
trematode, cestode, and protozoal infections)
Diagnose and manage common fungal infections
Clinically diagnose and manage common viral infections (caused by
common respiratory virus, herpesvirus, arbovirus)
Clinically diagnose and manage Infection in pregnancy (TORCH)

CURRICULUM CONTENT
1. The biology of infection: bacterial, viral, fungal and parasitic.
a. Principles of bacterial infections such as Staphylococci, Streptococci,
Neisseria, Salmonella, Vibrio, anaerobic bacteria Leptospira,
Mycobacteria, Gram positive bacilli)
b. Principles of viral infections such as respiratory virus (influenza virus,
mumps, measles), retrovirus (HIV), herpesvirus (HSV 1, HSV 2, VZV,
arbovirus (dengue virus, Japanese B encephalitis virus).
c. Principles of fungal infections such as Candida, Pneumocytis jiroveci,
Histoplasma, Cryptococcus
d. Principles of parasitic infections such as Plasmodium, Toxoplasma
gondii, Entamoeba histolytica and soil transmitted helminthes.
2. General approach to the patients with infection such as:
a. Clinical manifestations (local and systemic infections)
b. Laboratory examination to support diagnosis of infections i.e.
Microbiological examination, Parasites examination, Clinical pathology
examination, Pathology examination and Imaging examination

Udayana University Faculty of Medicine, DME, 2017 4


Study Guide Infection and Infectious Diseases

3. Management patients with infection such as:


a. Common bacterial infections such as bacterial meningitis, typhoid fever,
diarrhea, endocarditis, diphtheria, tetanus, food poisoning, genital
gonorrhoeae, non gonococcal urethritis, etc.
b. Common parasitic infections such as malaria, amoebiasis, toxoplasmosis.
c. Common fungal infection such as dermatophytosis, systemic candidiasis,
histoplasmosis, cryptococcosis, pneumocytis jiroveci pneumonia.
d. Common viral infections such as mumps, measles, influenza (especially
H5N1), SARS, varicella, herpes labialis, herpes genitalis, dengue fever,
Japanese B encephalitis, and HIV.
4. Immunization in children.
5. Infection in pregnancy

Udayana University Faculty of Medicine, DME, 2017 5


Study Guide Infection and Infectious Diseases

The Seven General Core Competencies

1. Patient Care
Demonstrate capability to provide comprehensive patient care that is compassionate,
appropriate, and effective for the management of health problems, promotion of health
and prevention of disease in the primary health care settings.

2. Medical Knowledge Base


Mastery of a core medical knowledge which includes the biomedical sciences,
behavioral sciences, epidemiology and statistics, clinical sciences, the social aspect of
medicine and the principles of medical ethics

3. Clinical skill
Demonstrate capability to effectively apply clinical skills and interpret the findings in
the investigation of the patients

4. Communication
Demonstrate capability to communicate effectively and interpersonally to establish
rapport with the patient, family, community at large, and professional associates, that
results in effective information exchange, the creation of a therapeutically and ethically
sound relationship

5. Information Management
Demonstrate capability to manager information which includes information access,
retrieval, interpretation, appraisal, and application to patiences specific problem, and
maintaining records of his or her proactive for analysis and improvement

6. Professionalism
Demonstrate a commitment to carrying out professional responsibilities and to
personal probity, adherence to ethical principles, sensitivity to a diverse patient
population and commitment to carrying out continual self-evaluation of his or her
professional standard and competence

7. Community-based and health system-based practice


Demonstrate awareness and responsiveness to larger context and system of health
care, and ability to effectively use system resource for optimal patient care.

Udayana University Faculty of Medicine, DME, 2017 6


Study Guide Infection and Infectious Diseases

PLANNERS TEAM

No Name Departement Phone


Prof. Dr. dr. Tuti Parwati Merati, Internal Medicine 08123806626
1 SpPD, KPTI-FINASIM (Tropical Disease)
Internal Medicine
2 dr. Made Susila Utama, SpPD-KPTI 08123815025
(Tropical Disease)
Dermatology & 081337808844
3 dr. Ratih Karna, Sp.KK
Venerology
Anatomy
4 dr. Ni Putu Ekawati, M.repro, Sp.PA 08113803933
Phatology
dr. I Gusti Ayu Harry Sundariyati,
5 DME 081805380277
S.Ked

LECTURER
NO NAME DEPT PHONE
1. Prof. Dr. dr. Tuti Parwati Merati, Internal Medicine 08123806626
SpPD, KPTI-FINASIM (Tropical Disease)
3. Internal Medicine
dr. Made Susila Utama, SpPD-KPTI 08123815025
(Tropical Disease)
4. Internal Medicine 08123989353
Dr. dr. Agus Somia, SpPD-KPTI
(Tropical Disease)
5. Internal Medicine 08123803985
dr. AA Yuli Gayatri, SpPD-KPTI
(Tropical Disease)
6. dr. Ni Made Dewi Dian Sukmawati, Internal Medicine 081805656501
Sp.PD (Tropical Disease)
7. dr W. Gustawan,M.Sc., Sp.A Pediatri 08123848241
8. dr. Made Bagiada, Sp.PD-KP Pulmonology 08123607874
9. Internal Medicine 08123914095
Dr. dr. Wayan Sudhana, SpPD-KGH
(Nefrology)
10. Obstetric & 081558101719
Dr. dr. IB Fajar Manuaba, SpOG
Gynecology
11. dr. Dwi Lingga Utama, Sp.A (K) Pediatric 081353002002
12. dr. Ni Made Susilawati, Sp.S (K) Neurology 08124690137
13. Prof. dr. Made Swastika Adiguna, Dermato &
08123828548
Sp.KK Venerology
14. Dermato & 081338645288
Dr. dr. A.A.G.P. Wiraguna, Sp.KK
Venerology
15. dr. Kadek Swastika, M.Kes Parasitology 08124649002
16. dr. Ni Nengah Dwi Fatmawati, Sp.MK,
Microchemistry 087862200814
Ph.D

Udayana University Faculty of Medicine, DME, 2017 7


Study Guide Infection and Infectious Diseases

FACILITATORS
(REGULAR CLASS)
Venue
No Name Group Departement Phone
(3rd floor)
3rd floor:
1 Dr. dr. Ni Made Linawati, M.Si A1 Histology 081337222567
R.3.01
dr. I Gusti Ayu Harry Sundariyati, 3rd floor:
2 A2 DME 081805380277 R.3.02
S.Ked
3rd floor:
3 dr. Yukhi Kurniawan, Sp.And A3 Andrology 08123473593 R.3.03
dr. Ni Nengah Dwi Fatmawati, 3rd floor:
4 A4 Microbiology 087862200814 R.3.04
Sp.MK, Ph.D
dr. Prima Sanjiwani Saraswati Dermatovener 3rd floor:
5 A5 08123818826
Sudarsa, M.Biomed, SpKK ology R.3.05
dr. Sri Yenni Trisnawati GS, 3rd floor:
6 A6 Neurology 081236223000
M.Biomed, Sp.S R.3.06
Prof. Dr.dr. I Putu Gede Adiatmika, 3rd floor:
7 A7 Physiology 08123811019 R.3.07
M.Kes
3rd floor:
8 dr. I Kadek Swastika, M.Kes A8 Parasitology 08124649002 R.3.08
3rd floor:
9 dr. I Wayan Surudarma, Msi A9 Biochemistry 081338486589
R.3.21
dr. Ni Made Ari Suryathi, 3rd floor:
10 A10 Opthalmology 085253651928
M.Biomed, Sp.M R.3.22
FACILITATORS
(ENGLISH CLASS)

Venue
No Name Group Departement Phone
(3rd floor)
3rd floor:
1 dr. Putu Cintya D.Y, MPH B1 Public Health 081353380666
R.3.01
Dr. dr. I Dewa Made Sukrama, Msi, 3rd floor:
2 B2 Microbiology 081338291965 R.3.02
Sp.MK (K)
3rd floor:
3 dr. I Ketut Wibawa Nada, Sp.An B3 Anestesiology 087860602995
R.3.03
dr. Ni Putu Ekawati, M.Repro, Anatomy 3rd floor:
4 B4 08113803933
Sp.PA Patology R.3.04
3rd floor:
5 Dr. dr. Susy Purnawati, M.KK B5 Physiology 08123989891 R.3.05
3rd floor:
6 Prof. dr. I G M. Aman, Sp.FK B6 Pharmacology 081338770650
R.3.06
3rd floor:
7 dr. Ni Putu Witari, Sp.S B7 Neurology 081338724040
R.3.07
3rd floor:
8 dr. I Wayan Sugiritama, M.Kes B8 Histology 08164732743 R.3.08
3rd floor:
9 Dr. dr. I Made Muliarta, M.Kes B9 Physiology 081338505350 R.3.21
3rd floor:
10 dr. Muliani, M.Biomed B10 Anatomy 085103043575
R.3.22

Udayana University Faculty of Medicine, DME, 2017 8


Study Guide Infection and Infectious Diseases

TIME-TABLE
(Block Infection and infectious Diseases)

DAY/DATE Time TOPIC Learning Place PIC


Situation
Regular English
Class Class
1 09.00- 08.00- Lecture 1: Introduction Class Dr Made Susila
Friday 09.30 08.30 Introduction of of block and room Utama, SpPD-KPTI
Oct 6th 2017 block infection and basic
infectious disease infection
09.30- 08.30- Lecture 2: Malaria Lecture Class Dr Made Susila
10.00 09.00 room Utama, SpPD-KPTI
12.00- 09.00- Individual
13.30 10.30 learning
13.30- 10.30- Small Group Disc. facilitator
15.00 12.00 Discussion Room
10.00- 12.30- Individual
11.30 14.00 learning
15.00- 14.00- Plenary Class
15.30 14.30 session room
15.30- 14.30- Plenary Class
16.00 15.00 session room
2 09.00- 08.00- Lecture 3: Dengue Lecture Class Dr Dewi Dian
Monday Oct 10.00 09.00 viral infection room Sukmawati, SpPD
9th 2017
12.00- 09.00- Individual
13.30 10.30 learning
12.00- 10.30- Small Group Disc. facilitator
13.30 12.00 Discussion Room
13.30- 12.30- Individual
15.00 14.00 learning
15.00- 14.00- Plenary Class Dr Dewi Dian
16.00 15.00 room Sukmawati, SpPD
3 09.00- 08.00- Lecture 4: Host Lecture Class Prof DR Dr Tuti
Tuesday 10.00 09.00 response to room Parwati Merati,
Oct 10th infection (viral, SpPD-KPTI
2017 bacterial, fungal
and parasite)
12.00- 09.00- Individual
13.30 10.30 learning
13.30- 10.30- Small Group Disc. Facilitator
15.00 12.00 Discussion room
10.00- 12.30- Individual
11.30 14.00 learning

Udayana University Faculty of Medicine, DME, 2017 9


Study Guide Infection and Infectious Diseases

15.00- 14.00- Plenary Class Prof DR Dr Tuti


16.00 15.00 room Parwati Merati,
SpPD-KPTI
4 09.00- 08.00- Lecture 5: Infection Lecture Class Dr Dwi Lingga
Wednesday 10.00 09.00 in Pediatric room Utama, SpA (K)
Oct 11th 12.00- 09.00- Individual
2017 13.30 10.30 learning
13.30- 10.30- Small Group Disc. Facilitator
15.00 12.00 Discussion room
10.00- 12.30- Individual
11.30 14.00 learning
15.00- 14.00- Plenary Class Dr Dwi Lingga
16.00 15.00 room Utama, SpA (K)
5 09.00- 08.00- Lecture 6: infection Lecture Class DR Dr IB Fajar
Thursday 10.00 09.00 in pregnancy room Manuaba, SpOG
Oct 12th 12.00- 09.00- Individual
2017 13.30 10.30 learning
13.30- 10.30- Small Group Disc. Facilitator
15.00 12.00 Discussion room
10.00- 12.30- Individual Class
11.30 14.00 learning room
15.00- 14.00- Plenary DR Dr IB Fajar
16.00 15.00 Manuaba, SpOG
6 09.00- 08.00- Lecture 7: Central Lecture Class Dr Ni Made
Friday 10.00 09.00 nervous system room Susilawati, SpS(K)
Oct 13th infection
2017 (meningitis,
encephalitis)
12.00- 09.00- Individual
13.30 10.30 learning
13.30- 10.30- Small Grup Disc. Facilitator
15.00 12.00 Discussion room
10.00- 12.30- Individual
11.30 14.00 learning
15.00- 14.00- Plenary Class Dr Ni Made
16.00 15.00 room Susilawati, SpS(K)
7 09.00- 08.00- Lecture 8: typhoid Lecture Class DR Dr Ketut Agus
Monday 09.30 08.30 fever room Somia, SpPD-KPTI
Oct 16th 09.30- 08.30- Lecture 9: Acute Lecture Class Dr AAA Yuli
2017 10.00 09.00 gastroenteritis room Gayatri, SpPD-KPTI
12.00- 09.00- Individual
13.30 10.30 learning
13.30- 10.30- Small Group Disc. Facilitator
15.00 12.00 Discussion room
10.00- 12.30- Individual Class
11.30 14.00 learning room

Udayana University Faculty of Medicine, DME, 2017 10


Study Guide Infection and Infectious Diseases

15.00- 14.00- Plenary Class DR Dr Ketut Agus


15.30 14.30 room Somia, SpPD-KPTI
15.30- 14.30- Plenary Dr AAA Yuli
16.00 15.00 Gayatri, SpPD-KPTI
8 09.00- 08.00- Lecture 10:HIV Lecture Class Prof DR Dr Tuti
Tuesday 10.00 09.00 infection: room Parwati Merati,
Oct 17th pathogenesis and SpPD-KPTI
2017 opportunistic
infections
12.00- 09.00- Individual
13.30 10.30 learning
13.30- 10.30- Small Group Disc. Facilitator
15.00 12.00 Discussion room
10.00- 12.30- Individual
11.30 14.00 learning
15.00- 14.00- Plenary Class Prof DR Dr Tuti
16.00 15.00 room Parwati Merati,
SpPD-KPTI
9 09.00- 08.00- Lecture 11: Lecture Class DR Dr Ketut Agus
Wednesday 09.30 08.30 Leptospirosis room Somia, SpPD-KPTI
Oct 18th
2017
09.30- 08.30- Lecture 12: Lecture Dr Dewi Dian
10.00 09.00 Filariasis Sukmawati, SpPD
12.00- 09.00- Individual
13.30 10.30 learning
13.30- 10.30- Small Group Disc. Facilitator
15.00 12.00 Discussion room
10.00- 12.30- Individual
11.30 14.00 learning
15.00- 14.00- Plenary Class DR Dr Ketut Agus
15.30 14.30 room Somia, SpPD-KPTI
15.30- 14.30- Plenary Class Dr Dewi Dian
16.00 15.00 room Sukmawati, SpPD
10 09.00- 08.00- Lecture 13: Child Lecture Class Dr Wayan
Thursday 09.30 08.30 Imunization room Gustawan, SpA(K)
Oct 19th
2017
09.30- 08.30- Lecture 14: Lecture Class Dr Made Susila
10.00 09.00 Bacteriemia, sepsis room Utama, SpPD-KPTI
12.00- 09.00- Individual
13.30 10.30 learning
13.30- 10.30- Small group Disc. Facilitator
15.00 12.00 discussion room
10.00- 12.30- Individual
11.30 14.00 learning
15.00- 14.00- Plenary Class Dr Wayan
15.30 14.30 room Gustawan, SpA(K)

Udayana University Faculty of Medicine, DME, 2017 11


Study Guide Infection and Infectious Diseases

15.30- 14.30- Plenary Class Dr Made Susila


16.00 15.00 room Utama, SpPD-KPTI
11 09.00- 08.00- Lecture 15: urinary Lecture Class DR Dr Wayan
Friday 09.30 08.30 tract infection room Sudhana, SpPD-
Oct 20th (acute KGH
2017 pyelonephritis,
lower urinary tract
infection)
09.30- 08.30- Lecture 16: Lower Lecture Class Dr Made Bagiada,
10.00 09.00 respiratory tract room SpPD-KP
infection
(pneumonia, acute
bronchitis)
12.00- 09.00- Individual
13.30 10.30 learning
13.30- 10.30- Small group Disc. Facilitator
15.00 12.00 discussion room
10.00- 12.30- Individual
11.30 14.00 learning
15.00- 14.00- Plenary Class DR Dr Wayan
15.30 14.30 room Sudhana, SpPD-
KGH
15.30- 14.30- Plenary Class Dr Made Bagiada,
16.00 15.00 room SpPD-KP
12 09.00- 08.00- Lecture 17: avian Lecture Class DR Dr Ketut Agus
Monday 09.30 08.30 influenza, SARS room Somia, SpPD-KPTI
Oct 23rd
2017
09.30- 08.30- Lecture 18: tetanus Lecture Dr AAA Yuli
10.00 09.00 Gayatri, SpPD-KPTI
12.00- 09.00- Individual
13.30 10.30 learning
13.30- 10.30- Small Group Disc. Facilitator
15.00 12.00 Discussion room
10.00- 12.30- Individual
11.30 14.00 learning
15.00- 14.00- Plenary Class DR Dr Ketut Agus
15.30 14.30 room Somia, SpPD-KPTI
15.30- 14.30- Plenary Class Dr AAA Yuli
16.00 15.00 room Gayatri, SpPD-KPTI
13 09.00- 08.00- Lecture 19: Skin Lecture Class
Prof. dr. Made
Tuesday 10.00 09.00 infection room
Swastika Adiguna,
Oct 24th
Sp.KK
2017
12.00- 09.00- Individual
13.30 10.30 learning
13.30- 10.30- Small Group Disc. Facilitator
15.00 12.00 Discussion room

Udayana University Faculty of Medicine, DME, 2017 12


Study Guide Infection and Infectious Diseases

10.00- 12.30- Individual


11.30 14.00 learning
15.00- 14.00- Plenary Class Prof. dr. Made
16.00 15.00 room Swastika Adiguna,
Sp.KK
14 09.00- 08.00- Lecture 20: S Lecture Class Dr. dr. A.A.G.P.
Wednesday 10.00 09.00 Sexually room Wiraguna, Sp.KK
Oct 25th Transmitted
2017 Infection

12.00- 09.00- Individual


13.30 10.30 learning
13.30- 10.30- Small Group Disc.
15.00 12.00 Discussion room
10.00- 12.30- Individual
11.30 14.00 learning
15.00- 14.00- Plenary Class
16.00 15.00 room
15 08.00- 08.00- STUDENT PROJECT Theat Evaluator/
Thursday 16.00 16.00 (REGULAR CLASS) er facilitator
Oct 26th room
2017 /Class
room
16 08.00- 08.00- STUDENT PROJECT Theat Evaluator/
Friday 16.00 16.00 (ENGLISH CLASS) er facilitator
Oct 27th room
2017 /Class
room
17 09.00- 08.00- Basic Clinical skill : Skill dr. Ni Made Dewi
Monday 16.00 15.00 Diagnostic of Dengue Viral Lab Dian Sukmawati,
Oct 30th Infection Sp.PD
2017

Vaccine Administration Skill dr W.


Lab Gustawan,M.Sc.,
Sp.A
18 09.00- 08.00- Basic Clinical skill : Skill DR Dr IB Fajar
Friday 16.00 15.00 Culture sampling technique in Lab Manuaba, SpOG
Nov 3rd 2017 gynecology
19 09.00- 08.00- Basic Clinical skill : dr. Kadek Swastika,
Monday 16.00 15.00 Fecal Examination M.Kes
Nov 6th 2017
20 09.00- 08.00- Basic Clinical skill : dr. Ni Nengah Dwi
Tuesday 16.00 15.00 Fatmawati, Sp.MK,
Nov 7th 2017 Ph.D
Wednesday PRE-EVALUATION BREAK
Nov 08th
2017
Thursday EVALUATION
Nov 09th

Udayana University Faculty of Medicine, DME, 2017 13


Study Guide Infection and Infectious Diseases

STUDENT PROJECT

Regulation:
TITLE
Name:
NIM:
Faculty of Medicine, Udayana University
2016

1. Introduction (Pendahuluan)
2. Content (Isi sesuai dengan judul paper)
3. Summary (Ringkasan)
4. References (Daftar pustaka): Vancouver style
5. Pages: 6-10, Spasi: 1.5, Time New Roman:12

TOPIC Regular Class (Class A)


GROUP DATE & TITLE PEMBIMBING/ EVALUATOR
TIME FASILITATOR
1 Tuesday, Cerebral abses dr. Ni Made
Sept 20th Susilawathi,
2016, Sp.S (K)
08.30-09.00
2 09.00-09.30 Cerebral
toxoplasmosis
3 09.30-10.00 Multi drug dr. Made
resisten (MDR) Bagiada,
TB Sp.PD-KP
4 10.00-10.30 Spondilitis TB
5 10.30-11.00 Acute infection dr. A.A.A. Yuli
pancreatitis Gayatri, Sp.PD-
6 11.00-11.30 Pes KPTI
11.30-12.30 BREAK
7 12.30-13.00 Leishmaniasis dr. Dewi Dian
8 13.00-13.30 Tripanosomiasis Sukmawati,
Sp.PD
9 13.30-14.00 Cytomegalovirus Dr. dr. Ketut
infection Agus Somia,
10 14.00-14.30 Squamous cell Sp.PD-KPTI
carcinoma
(associated with
HPV infection)

Udayana University Faculty of Medicine, DME, 2017 14


Study Guide Infection and Infectious Diseases

TOPIC English Class (Class B)


GROUP DATE & TIME TITLE PEMBIMBING/ EVALUATOR
FASILITATOR
1 Wednesday, Nasopharyngeal dr. Wayan
th
Oct 5 2016, carcinoma Gustawan,
08.30-09.00 (associated with EBV Sp.A (K)
infection)
2 09.00-09.30 Hairy leukoplakia
(associated with EBV
infection)
3 09.30-10.00 Korioretinitis dr. Ratih
4 10.00-10.30 Carcinoma servik Karna, Sp.KK
(associated with HPV
infection)
5 10.30-11.00 Penyakit jantung dr. Dwi
rematik Lingga
6 11.00-11.30 Endoftalmitis Utama, Sp.A
(K)
11.30-12.30 BREAK
7 12.30-13.00 Hepatitis C dr. Made
8 13.00-13.30 Abses payudara Susila Utama,
Sp.PD-KPTI
9 13.30-14.00 Epididimitis Dr. dr. I.B.
10 14.00-14.30 Herpes genital tipe 2 Fajar
Manuaba,
Sp.OG

Udayana University Faculty of Medicine, DME, 2017 15


Study Guide Infection and Infectious Diseases

ASSESSMENT METHOD

1. Assessment will be held twice, for infection & infectious diseases. Final mark is
combination beteween this mark. The time provision is 100 minutes. The number of
MCQ is 100 with passing point 70.
2. Assessment in this block consists of:
SGD : 5%
Student Project (Paper) : 10%
Exam : 85%

Udayana University Faculty of Medicine, DME, 2017 16


Study Guide Infection and Infectious Diseases

LEARNING PROGRAM

Lecture 1
INTRODUCTION OF BLOCK INFECTION AND INFECTIOUS
DISEASE

Abstract
Infections and infectious diseases are a great burden on many societies, including
Indonesia. To reduce that burden an integrated approach is required, combining
health promotion, disease prevention and patient treatment. The prerequisite for
success in this fight is the participation of all health care professionals. Should know
and understand terminology commonly use in the context of infectious disease.
Infectious diseases are disorders or diseases caused by organisms such as
bacteria, viruses, fungi or parasites. Many organisms live in and on our bodies.
They're normally harmless or even helpful, which we called them normal flora; but
under certain conditions, some organisms may cause disease. This organisms
called as pathogen as they can produced pathology to the body.
Infectious diseases are one of the leading causes of death worldwide. Infectious
diseases can be spread directly or indirectly. Some infectious diseases can be
passed from person to person. Some are transmitted by bites from insects or
animals. And others are acquired by ingesting contaminated food or water or being
exposed to organisms in the environment.
Many infectious diseases become difficult to control if the infectious agents evolve
resistance to commonly used drugs: For example, bacteria can accumulate
mutations in their DNA or acquire new genes that allow them to survive contact with
antibiotic drugs that would normally kill them.
Signs and symptoms vary depending on the organism causing the infection, but
often include fever and fatigue. Mild infections may respond to rest and home
remedies, while some life-threatening infections may require hospitalization.
Many infectious diseases, such as measles and chickenpox, can be prevented by
vaccines but many other still do not have vaccine available. Other prevention mean
such as frequent and thorough hand-washing helps protect from most infectious
diseases.

Symptoms
Each infectious disease has its own specific signs and symptoms. General signs
and symptoms common to a number of infectious diseases include: fever, fatigue,
muscle aches, coughing and diarrhea
Transmission: an infection can be spread through direct and indirect contact
Direct contact
An easy way to catch most infectious diseases is by coming in contact with a person
or animal who has the infection. Three ways infectious diseases can be spread
through direct contact are:
Person to person. A common way for infectious diseases to spread is through the
direct transfer of bacteria, viruses or other germs from one person to another. This
can occur when an individual with the bacterium or virus touches, kisses, or coughs
or sneezes on someone who isn't infected. These germs can also spread through

Udayana University Faculty of Medicine, DME, 2017 17


Study Guide Infection and Infectious Diseases

the exchange of body fluids from sexual contact. The person who passes the germ
may have no symptoms of the disease, but may simply be a carrier.
Animal to person. Being bitten or scratched by an infected animal even a pet
can make you sick and, in extreme circumstances, can be fatal. Handling animal
waste can be hazardous, too. For example, you can acquire a toxoplasmosis
infection by scooping your cat's litter box.
Mother to unborn baby. A pregnant woman may pass germs that cause infectious
diseases to her unborn baby. Some germs can pass through the placenta. Germs in
the vagina can be transmitted to the baby during birth.
Indirect contact
Touching: Disease-causing organisms also can be passed by indirect contact.
Many germs can linger on an inanimate object, such as a tabletop, doorknob or
faucet handle. When you touch a doorknob handled by someone ill with the flu or a
cold, for example, you can pick up the germs he or she left behind. If you then touch
your eyes, mouth or nose before washing your hands, you may become infected.
Vectors/insect bites - Some germs rely on insect carriers such as mosquitoes,
fleas, lice or ticks to move from host to host. These carriers are known as
vectors. Mosquitoes can carry the malaria parasite or West Nile virus, and deer ticks
may carry the bacterium that causes Lyme disease.
Food/water contamination : disease-causing germs can infect you is through
contaminated food and water. This mechanism of transmission allows germs to be
spread to many people through a single source. E. coli, for example, is a bacterium
present in or on certain foods such as undercooked hamburger or unpasteurized
fruit juice.

Risk factors
Anyone can catch infectious diseases easier than other people because the body
immune system is not work well. This may occur if there are primary or secondary
immune deficiency , such as : Taking steroids or other medications that suppress
immune system, such as anti-rejection drugs for a transplanted organ, certain types
of cancer or other disorders that affect the immune system, infection by HIV or
AIDS. In addition, certain other medical conditions may predispose you to infection,
including implanted medical devices, malnutrition and extremes of age, among
others.

Complications
Most infectious diseases have only minor complications. But some infections
such as pneumonia, AIDS and meningitis can become life-threatening. A few
types of infections have been linked to a long-term increased risk of cancer:
hepatitis B and C have been linked to liver cancer, human papillomavirus is linked to
cervical cancer, Helicobacter pylori is linked to stomach cancer and peptic ulcers
and In addition, some infectious diseases may become silent, only to appear again
in the future sometimes even decades later. For example, someone who's had a
chickenpox infection may develop shingles much later in life.

Diagnosis
Diagnosis can be made with clinical symptoms and signs. Because many of
infectious disease have a very similar symptoms and signs, usually laboratory or
imaging test would be needed to confirmed the diagnosis
Laboratory tests

Udayana University Faculty of Medicine, DME, 2017 18


Study Guide Infection and Infectious Diseases

Many infectious diseases have similar signs and symptoms. Samples of body fluids
can sometimes reveal evidence of the particular microbe that's causing illness.
Samples can get from blood, urine, throat swabs, stool sample, spinal tap (lumbar
puncture). A technician obtains a sample with a standard procedure from each.
Imaging scans
Imaging procedures such as X-rays, computerized tomography and magnetic
resonance imaging can help pinpoint diagnoses and rule out other conditions that
may be causing the symptoms.
Biopsies
During a biopsy, a tiny sample of tissue is taken from an internal organ for testing.
For example, a biopsy of lung tissue can be checked for a variety of fungi that can
cause a type of pneumonia.

Treatment
Knowing what type of germ is causing the illness makes it easier to choose
appropriate treatment such as : antibiotics, anti viral, anti fungal, and anti-
parasitics.
However, the use of those agents should be use appropriately. The overuse of
antibiotics has resulted in several types of bacteria developing resistance to one or
more varieties of antibiotics. This makes these bacteria much more difficult to treat
As an alternative medicine, a number of products have been purported to help fend
off common illnesses, such as the cold or flu. Cranberry, echinacea, garlic, ginseng,
vitamin C, D and zinc are among other. While some of these substances have
appeared promising in early trials, follow-up studies may have had negative or
inconclusive results. More research needs to be done.

Prevention
Infectious diseases can be prevented by vaccines but many other still do not have
vaccine available. Prevention to infectious disease transmission with no vaccine
available should use another approach. Personal protected equipment such as
gown, masker, google, gloves, and shoes boot. Infectious disease transmitted by
vectors should avoid to being bitten by mosquitoes or other insects may be useful,
such as wearing long sleeve shirt, and putting mosquitos repellent. Other
prevention means such as frequent and thorough hand-washing helps protect
from most infectious diseases.

Learning tasks:
1. How do we know that a patient in our hospital is suffering from a kind of
infectious disease?
2. How he or she can get it?
3. What prevention do we need to do to avoid of getting any infectious
diseases?

Self assessment:
1. Describe all common terminology use in the context of infectious diseases.
2. Describe general clinical manifestation of infectious diseases.
3. Describe pathogenesis of general symptoms of infectious disease.

Udayana University Faculty of Medicine, DME, 2017 19


Study Guide Infection and Infectious Diseases

Lecture 2
MALARIA

Abstract
Malaria is caused by infection of red blood cells with protozoan parasites of
the genus Plasmodium inoculated into the human host by a feeding female
anopheline mosquito. The five human Plasmodium species transmitted from person
to person are P. falciparum, P. vivax, P. Ovale (two species), P. Malariae and
P. Knowlesi. The first symptoms of malaria are nonspecific and similar to those of a
minor systemic viral illness. They comprise headache, lassitude, fatigue, abdominal
discomfort and muscle and joint aches, usually followed by fever, chills, perspiration,
anorexia, vomiting and worsening malaise. Disease progression to severe malaria
may take days but can occur within a few hours. Severe malaria usually manifests
with one or more of the following: coma (cerebral malaria), metabolic acidosis,
severe anaemia, hypoglycaemia, acute renal failure or acute pulmonary oedema. If
left untreated, severe malaria is fatal in the majority of cases. All patients with
suspected malaria should be treated on the basis of a confirmed diagnosis by
microscopy examination or RDT testing of a blood sample. Management of
uncomplicated malaria using ACT (Artemisinine Combination Therapy) for 3 days
but severe malaria must treated with anti malaria intravenous (artesunate).

Learning task
1. Explain clinical symptom and sign of malaria
2. Differentiate between uncomplicated malaria and severe malaria
3. Management uncomplicated malaria and severe malaria

Self assessment
Case:
1. Male, 23 years old come to internal medicine clinic with fever for 6 days.
Fever was intermittenly every 2 days, fever was followed by chills and
sweathing. He has history travel to Papua.
- How to make diagnosis in this case?
- How to manage this case?
2. Female, 34 years old come to emergency room with severe headache and
fever for 4 days. Physical examination: icterus +, lien S2. Lab examination Hb
4 g/dL, bil total 4,3 mg/dL and plasmodium falciparum on smear.
- What is the assessment?
- How to manage this case?

Udayana University Faculty of Medicine, DME, 2017 20


Study Guide Infection and Infectious Diseases

Lecture 3
DENGUE FEVER, DENGUE HEMORRHAGIC FEVER, DENGUE
SHOCK SYNDROME

ABSTRACT
Dengue is mosquito borne viral infection, which causing acute fever and
occasionally develops into potential lethal complication. Dengue virus is transmitted
by female mosquitoes mainly of the species Aedes aegypti and, to a lesser
extent, Ae. albopictus. This mosquito also transmits Chikungunya, Yellow Fever and
Zika infection. About half of world population were at risk, Dengue is widespread
throughout the tropics, with local variations in risk influenced by rainfall, temperature
and unplanned rapid urbanization.
There are 4 distinct, but closely related, serotypes of the virus that can cause
dengue (DEN-1, DEN-2, DEN-3 and DEN-4). Recovery from infection by one
provides lifelong immunity against the particular serotype with partial and temporary
cross-immunity to the other serotypes after recovery. Subsequent infections by other
serotypes increase the risk of developing severe dengue.
Dengue should be suspected when a high fever is present, accompanied by
2 of the following symptoms: severe headache, pain behind the eyes, muscle and
joint pains, nausea, vomiting, swollen glands or rash. Symptoms usually last for 27
days, after an incubation period of 4 5 days (range 3 14 days) after the bite from
an infected mosquito. Severe dengue is a potentially deadly complication due to
plasma leakage, fluid accumulation, respiratory distress, severe bleeding, or organ
impairment. Warning signs occur 37 days after the first symptoms in conjunction
with a decrease in temperature and include: severe abdominal pain, persistent
vomiting, and rapid breathing, bleeding gums, fatigue, restlessness and blood in
vomit. The next 2448 hours of the critical stage can be lethal; proper medical care
is needed to avoid complications and risk of death.
For the purpose of clinical management, WHO classifies dengue illness as (i)
dengue with or without warning signs for progression towards severe dengue and (ii)
severe dengue. Warning signs of severe dengue include abdominal pain or
tenderness, persistent vomiting, clinical fluid accumulation, mucosal bleeding,
lethargy or restlessness, liver enlargement of >2 cm, or an increase in Hematocrit
concurrent with a rapid decrease in platelet count. Criteria for severe dengue include
any sign of severe plasma leakage leading to shock or fluid accumulation with
respiratory distress, severe bleeding, or severe organ impairment.
There is no specific treatment for dengue, maintenance of the patients body
fluid volume is critical in management of severe dengue. Vaccine for prevention is
now commercially available, Dengvaxia (CYD TDV) for use in individual age 9
45 years living in area with high burden of disease.

LEARNING TASK
Case 1:
IMD, 16 year old male student come to a private clinic with acute onset of high fever.
The fever starts abruptly one day before accompanied by nausea and retro orbital
pain.
Questions:

Udayana University Faculty of Medicine, DME, 2017 21


Study Guide Infection and Infectious Diseases

1. What other information needed in anamnesis of this patient? What data


should be obtained during physical examination?
2. The evaluation shows vital sign was stabile without any sign of bleeding, the
patient able to drink plenty and eat normally. What is the diagnosis of this
patient? How about further plan for this case, including planning for
diagnostic, treatment, monitoring and patient and familys education?
3. The next day, he showed his laboratory evaluation. WBC: 1.03 x 10 3/uL. Hb.
17 g/dL. HCT 54 %. Plt. 98 x 10 3/uL. What is the diagnosis of this patient?
How about further plan for this case, including planning for diagnostic,
treatment, monitoring and patient and familys education?

Case 2:
25 year old woman, working as a private secretary, come to the clinic with 4 days of
fever. Since morning she also had her menstrual bleeding which comes 2 weeks
early. The initial evaluation revealed moderately ill, fully alert. Blood pressure 90/76
mmHg. Pulse rate 124 x/minutes. Respiratory rate: 22 x/minutes. Temperature
axillae: 36.7C. Hepatomegaly 3 cm below costal arch, tender on palpation, the
acrals were cold. She has her laboratory evaluation: WBC: 3.02 x 10 3/uL; Hb. 16
g/dL. HCT: 51.2 %. Plt.32 x 103/uL
1. What is the diagnosis of this patient? How about further plan for this case,
including planning for diagnostic, treatment, monitoring and patient and
familys education?
2. Is there any indication for platelet transfusion in this case? Why?
3. The patient was then hospitalized, by the third day of her hospital stay, she
complain about shortness of breath and cough if she change position from
supine to sitting. The physical evaluation revealed dullness on lower part of
right hemithorax with decreased breath sounds. What kind of complication
can be expected in severe dengue cases? How can we establish the
diagnosis?

Self assessment
1. How can we differentiate between dengue fever and dengue hemorrhagic
fever? Make a comparison chart on this issue!
2. Both DHF grade 3 and 4 are included as DSS. How can we differentiate
between the two of them?
3. What is NS1 dengue antigen? When is the best way to use it?
4. How about dengue serology? When IgM anti-Dengue become positive?
When the Ig G anti-dengue become positive? Make the chart on the possible
result and its interpretation!

REFERENCES:
1. World Health Organization. Dengue Guidelines for Diagnosis, Treatment,
Prevention and Control. Geneva, Switzerland: 2009. Available at
http://www.who.int/tdr/ publications/documents/dengue-diagnosis.pdf
2. Messina JP, et al. Global spread of dengue virus types: mapping the 70 year
history. Trends Microbiol. 2014;22(3):138146.
3. Guzman MG, et al. Dengue. Lancet. 2015;385(9966):453465.
4. Comprehensive guidelines for prevention and control of dengue and dengue
hemorrhagic fever. WHO SEARO 2011
5. Handbook for clinical management of dengue. WHO 2012

Udayana University Faculty of Medicine, DME, 2017 22


Study Guide Infection and Infectious Diseases

Lecture 4
HOST RESPONSE TO INFECTION

Abstract
Infectious diseases are one of the leading causes of death worldwide. Many
infectious diseases become difficult to control if the infectious agents evolve
resistance to commonly used drugs, for example, bacteria can accumulate
mutations in their DNA or acquire new genes that allow them to survive contact with
antibiotic drugs that would normally kill them. Scientists are currently searching for
new approaches to treat infectious diseases, focusing on exactly how the pathogens
change and drug resistance evolves. Understanding and comprehend the host
response to infection is important.
Infection process
After invading the body, microorganisms must multiply to cause infection. After
multiplication begins, one of three things can happen:
1. Microorganisms continue to multiply and overwhelm the bodys defenses.
2. A state of balance is achieved, causing chronic infection.
3. The bodywith or without medical treatmentdestroys and eliminates the
invading microorganism.
Defenses Against Infection
Host defenses that protect against infection :
Natural barriers (eg, skin, mucous membranes)
Nonspecific immune responses (eg, phagocytic cells [neutrophils,
macrophages] and their products)
Specific immune responses (eg, antibodies, lymphocytes)
Nonspecific Immune Responses
Cytokines (including IL-1, IL-6, tumor necrosis factor-alpha, and interferon-
gamma) are produced principally by macrophages and activated lymphocytes
and mediate an acute-phase response that develops regardless of the
inciting microorganism.
The response involves fever and increased production of neutrophils by the
bone marrow. Endothelial cells also produce large amounts of IL-8, which
attracts neutrophils.
The inflammatory response directs immune system components to injury or
infection sites and is manifested by increased blood supply and vascular
permeability, which allows chemotactic peptides, neutrophils, and
mononuclear cells to leave the intravascular compartment.
Specific Immune Responses
After infection, the host can produce a variety of antibodies (complex
glycoproteins known as immunoglobulins) that bind to specific microbial
antigenic targets. Antibodies can help eradicate the infecting organism by
attracting the hosts WBCs and activating the complement system.
The complement system destroys cell walls of infecting organisms, usually
through the classical pathway. Complement can also be activated on the
surface of some microorganisms via the alternative pathway.
Antibodies can also promote the deposition of substances known as opsonins
(eg, the complement protein C3b) on the surface of microorganisms, which
helps promote phagocytosis. Opsonization is important for eradication of
encapsulated organisms such as pneumococci and meningococci.

Udayana University Faculty of Medicine, DME, 2017 23


Study Guide Infection and Infectious Diseases

Interaction between host and the microbes are depend on factors from the host,
the microbes and environment
Host Genetic Factors
For many pathogens, the host's genetic make-up influences the host's
susceptibility and the resulting morbidity and mortality. For example, patients
who have deficiencies of the terminal complement components (C5 through
C8, perhaps C9) have an increased susceptibility to infections caused by
neisserial species.
Factors Facilitating Microbial Invasion
Microbial invasion can be facilitated by the
following:
Virulence factors
Microbial adherence, biofilm
Resistance to antimicrobials
Defects in host defense mechanisms
Defects in Host Defense Mechanisms
Two types of immune deficiency states affect the hosts ability to fight infection:
Primary immune deficiency : are genetic in origin; > 100 primary immune
deficiency states have been described. Most primary immune deficiencies are
recognized during infancy; however, up to 40% are recognized during
adolescence or adulthood.
Secondary (acquired) immune deficiency : are caused by another disease
(eg, cancer, HIV infection, chronic disease) or by exposure to a chemical or
drug that is toxic to the immune system.

Defects in immune responses may involve


Cellular immunity
Humoral immunity
Phagocytic system
Complement system
Mechanism
Cellular deficiencies are typically T-cell or combined immune defects. T
cells contribute to the killing of intracellular organisms; thus, patients with T-
cell defects can present with opportunistic infections such as Pneumocystis
jirovecii or cryptococcal infections. Chronicity of these infections can lead to
failure to thrive, chronic diarrhea, and persistent oral candidiasis.
Humoral deficiencies are typically caused by the failure of B cells to make
functioning immunoglobulins. Patients with this type of defect usually have
infections involving encapsulated organisms (eg, H. influenzae, streptococci).
Patients can present with poor growth, diarrhea, and recurrent sinopulmonary
infections.
A defect in the phagocytic system affects the immediate immune response
to bacterial infection and can result in development of recurrent abscesses or
severe pneumonias.
Primary complement system defects are particularly rare. Patients with this
type of defect may present with recurrent infections with pyogenic bacteria
(eg, encapsulated bacteria, Neisseria sp) and have an increased risk of
autoimmune disorders (eg, SLE).
Clinical manifestation

Udayana University Faculty of Medicine, DME, 2017 24


Study Guide Infection and Infectious Diseases

Most infections increase the pulse rate and body temperature, but others (eg,
typhoid fever, tularemia, brucellosis, dengue) may not elevate the pulse rate
commensurate with the degree of fever.
Hypotension can result from hypovolemia, septic shock, or toxic shock.
Hyperventilation and respiratory alkalosis are common.
Alterations in sensorium (encephalopathy) may occur in severe infection
regardless of whether CNS infection is present. Encephalopathy is most
common and serious in the elderly and may cause anxiety, confusion,
delirium, stupor, seizures, and coma.
Infectious diseases commonly increase the numbers of mature and immature
circulating neutrophils. Mechanisms include demargination and release of
immature granulocytes from bone marrow, IL-1- and IL-6-mediated release of
neutrophils from bone marrow, and colony-stimulating factors elaborated by
macrophages, lymphocytes, and other tissues. Exaggeration of these
phenomena (eg, in trauma, inflammation, and similar stresses) can result in
release of excessive numbers of immature leukocytes into the circulation
(leukemoid reaction), with leukocyte counts up to 25 to 30 10 9/L.
Conversely, some infections (eg, typhoid fever, brucellosis) commonly cause
leukopenia. In overwhelming, severe infections, profound leukopenia is often
a poor prognostic sign.
Characteristic morphologic changes in the neutrophils of septic patients
include Dhle bodies, toxic granulations, and vacuolization.
Anemia can develop despite adequate tissue iron stores. If anemia is chronic,
plasma iron and total iron-binding capacity may be decreased. Serious
infection may cause thrombocytopenia and disseminated intravascular
coagulation (DIC).

Learning Tasks:
A 40 years-old female admitted to hospital with decreased of consciousness. She
suffering from high fever since a week ago, and she also did not eat and drink
much as for her weaknesses. She had headache, sometimes vomits and feel
abdominal discomfort.
1. What is the working diagnosis of the patient?
2. What kind of examination do you need to do for this patient?
3. Please explain if there is a possibility of a defective imune response in this
patient?

Self Assessment:
1. Describe the host response to infection in this patient.
2. Based on the pathogenesis, please describe the management of the patient.

Udayana University Faculty of Medicine, DME, 2017 25


Study Guide Infection and Infectious Diseases

Lecture 5
INFECTION IN PEDIATRIC

1. DENGUE HEMORRHAGIC FEVER


References :
TatalaksanaDemam Dengue/DemamBerdarah Dengue
Krugmans Infectious Diseaseof Children
Nelson text book of Pediatrics

Abstract
Dengue hemorrhagic Fever (DHF) is a viral infection disease caused by dengue
viral (DEN 1, DEN 2, Den 3 and DEN 4) with the symptom that are : fever 2 to 7
days, bleeding manifestation, enlargement of liver and shock with laboratory
criteria thrombocytopenia and haemoconcentration. Four grade DHF from WHO
are : grade I; fever with no specific symptom & Rumple Leed (RL) +, grade II; grade
I + spontaneous bleeding, grade III; manifestation of circulatory failure and grade IV;
profound shock.
Management DHF according of the grading of disease as well as suspicion DHF,
DHF grade I, II and Dengue Shock Syndrome.

Learning Task

3 years old boys came with chief complaint high fever no decrease from three days
ago, no cough and without sore throat. According to his parent he had vomited, no
appetite no drink.

Task :
1. What are other helpful information you trying to get from his parent ?
2. What the physical examine you need to this patient ?
3. When the Rumple Leedepositive, what the kind of the laboratory test you
have done ?
4. If the laboratory test showed leucopenic and trombositopenic, how about the
management to this patient ?

Self Assessment:

1. What the clinical spectrum of dengue infection? Explain it !


2. How about the patophysiology of Dengue Hemorrhagic Fever ?
3. Explain the pathogenesis of shock in DHF !
4. How the bleeding occur in DHF ?
5. Explain DHF clinicallyaccording the WHOcriteria !
6. Explain the DHF complication !
7. How about the management of DHF? Explain it !
8. Mention the hospitalization of DHF patient (the indication of hospitalization)
9. Indication of discharged from hospital.

Udayana University Faculty of Medicine, DME, 2017 26


Study Guide Infection and Infectious Diseases

2. DIPHTHERIA
References :
Krugmans Infectious Diseaseof Chlidren
Nelson text book of Pediatrics
Nelson Essentials of Pediatrics

Abstract
Diphtheria is a preventable acute disease caused by Corynebacterium Diphtheria.
Diphtheria is acquired by contact with ill person with the disease or with an
asymptomatic carrier of organism. The incidence increase among school age
children during the first few month of the school year has been reported. Diphtheria
usually develops after a short incubation period of 2 days with a range of 1 to 5
days. For clinical purposes, the disease my be classified by the anatomic location as
a nasal diphtheria, tonsilar and pharyngeal diphtheria, laryngeal diphtheria,
cutaneus diphtheria and unusual type of diphtheria. The diagnosis of diphtheria is
confirmed by the demonstration of diphtheria bacilli cultured from material obtained
from the site of infection. Diphtheria antitoxin must be given promptly and in
adequate dosage. Penicillin and erythromycin are effective against most strains of
diphtheria bacilli.
Supportive therapy is needed for several cases and complication.
Preventive therapy by immunization with diphtheria toxoid.

Self Assessment

1. Explain the etiology of diphtheria !


2. How the pathogenesis of diphtheria ?
3. Mention the sign and symptom that we could find in patient suffered from
diphtheria based on the location of the infection !
4. Explain how to diagnosis diphtheria !
5. Mention the differential diagnosis of diphtheria based on the anatomic site of
involvement !
6. Explain some severe complication caused by the effect of diphtheria toxin
7. Explain the management of diphtheria !

3. TETANUS

References :
Krugmans Infectious Diseaseof Children
Nelson text book of Pediatrics
Nelson Essentials of Pediatrics

Abstract
Tetanus is an acute, spastic paralytic illness caused by tetanus toxin; the neurotoxin
produced by Clostridium Tetani. Tetanus occurs after induced spores germinate,
multiply and produce tetanus toxin at infected injury site. Tetanus toxin binds at the
neuromuscular junction and enter the motor nerve by endocytosis.
Clinical manifestation of tetanus is trismus, headache, restlessness, often followed
by stiffness, difficulty chewing, dysphagia and neck muscle spasm. Because tetanus
toxin does not affect sensory nerve, the patient remains conscious. The seizures are

Udayana University Faculty of Medicine, DME, 2017 27


Study Guide Infection and Infectious Diseases

characterized by sudden, severe tonic contractions of the muscle. Diagnosis is


established clinically with typical is an unimmunized patient who was injured or born
within the preceding 2 week who presents with trismus, other rigid muscle and a
clear sensorium. Management of tetanus requires eradication of C. tetani and the
wound environment conductive to its anaerobic multiplication, neutralization of
tetanus toxin, control of seizures and respiration, palliation and provision of
meticulous supportive care. Penicillin G remains the antibiotic of choice for C.
tetani, erythromycin and tetracycline (for patient 8 yr old or older) are alternatives for
penicillin allergic patients. Muscle relaxants for relaxation and seizure control.
Active immunization for prevent is tetanus toxoid usually combine with diphtheria
and pertusis vaccine.

Self assessment

1. How is the characteristic of Clostridium tetani ?


2. Explain about portdentre of the bacteria !
3. How is the pathogenesis of stiffness caused by the effect of toxin in
Tetanus ?
4. Mention the degree of tetanus and explain it !
5. Mention the differential diagnosis of Tetanus !
6. How is the management of patient with Tetanus ?
7. Explain some ways to prevent Tetanus!

4. MEASLES
References :
Krugmans Infectious Diseaseof Children
Nelson text book of Pediatrics
Nelson Essentials of Pediatrics

Abstract
Measles is an acute, highly contagious viral disease characterized by fever, coryza,
conjunctivitis, cough and a specific enanthem followed by a generalized
maculopapular eruption, caused by Morbillivirus, family Paramyxoviridae.
After an incubation period (10 to 11 days) initial symptom is fever and malaise;
within 24 hours there is onset of coryza, conjunctivitis and cough. The symptoms
gradually increase in severity with the appearance of eruption on fourth day and
kopliks spot on 2 days before rash. Anorexia and malaise are usually present during
the febrile period.
Measles is a self limiting disease therefore management is supportive therapy. The
preventive with measles vaccination.

Self assessment

1. How the pathologic lesions process when the measles infection occurs ?
2. Explain the schematic diagram of clinical course of typical case of measles!
3. What is the etiology of measles?
4. How is the pathogenesis of measles?
5. Explain the clinical manifestation that you could find in child who suffered
from measles!

Udayana University Faculty of Medicine, DME, 2017 28


Study Guide Infection and Infectious Diseases

6. Explain the management of patient with measles!


7. Explain how to prevent child from measles!

5. EMERGING DISEASE :AVIAN INFLUENZA


References :
Avian Influenza WHO guideline

Abstract
Avian influenza (AI) is an infectious disease of birds by type A strain of the influenza
virus. Mutation and reassortment (antigenic drift, antigenic shift) event are
commonly observed in the affected birds populations. Since 2003, the widespread
ongoing epizootic of AI A (H5N1) among poultry and birds has resulted in human
H5N1 cases in 10 countries. The first cases of H5N1 virus infection in Indonesia was
identified in July 2005. The reported symptoms of AI in humans have range from
typical influenza like symptoms (eq, fever, cough, sore throat, and muscle aches) to
eye infections (conjunctivitis), pneumonia, and other severe and live threatening
complication. Patient was diagnosed confirmed case when we found positive viral
culture for influenza A (H5N1) or positive for influenza (H5) or a four fold rise in H5
specific. Two drugs oseltamivir (Tamiflu ) and zanamivir (Relanza ) can reduce the
severity and duration of illness.

Self Assessment

1. What is the etiology of Avian Influenza ?


2. What the clinical symptom human who infected avian viruses ?
3. How do people become infected ?
4. How can we safe to consumption the poultry product ?
5. Explain the pandemic hazard this virus !
6. Explain the management of patient with Avian Influenza ?

Lecture 6
TORCH INFECTION IN PREGNANT WOMEN

Abstract
Infections have historically been a major cause of maternal andfetal morbidity and
mortality worldwide, and they remain soin the 21st century. The unique maternal-
fetal vascular connectionin some cases serves to protect the fetus from
infectiousagents, whereas in other instances it provides a conduitfor their
transmission to the fetus.TORCH infectionsinclude infections associated with
Toxoplasma, Other organisms (Parvovirus, human immunodeficiencyvirus, Epstein-
Barr virus, herpesviruses 6 and 8, varicella, syphilis, enteroviruses),
Rubella,Cytomegalovirus (CMV), and Hepatitis. Despite the recent emphasis in the
screening, antibioticprophylaxis, and management of early-onset many neonates
and children yearly experience the consequences of classic perinatal infections

Introduction
Infections have historically been a major cause of maternal and fetal morbidity and
mortality worldwide, and they remain so in the 21st century. The unique maternal-

Udayana University Faculty of Medicine, DME, 2017 29


Study Guide Infection and Infectious Diseases

fetal vascular connection in some cases serves to protect the fetus from infectious
agents, whereas in other instances it provides a conduit for their transmission to the
fetus.Maternal serological status,gestational age at the time infection is acquired,
the mode ofacquisition, and the immunological status of both the motherand her
fetus all influence disease outcome.

TORCH is an acronym for a groupof congenitally acquired infectionsthat may cause


significant morbidityand mortality in neonates. TORCH infections include infections
associated with Toxoplasma, Other organisms (Parvovirus, human
immunodeficiency virus, Epstein-Barr virus, herpesviruses 6 and 8, varicella,
syphilis, enteroviruses), Rubella, Cytomegalovirus (CMV), and Hepatitis. Despite the
recent emphasis in the screening, antibiotic prophylaxis, and management of early-
onset many neonates and children yearly experience the consequences of classic
perinatal infections.

Toxoplasmosis
The obligate intracellular parasite Toxoplasma gondii has a lifecycle with two distinct
stages. The feline stage takesplace in the catthe definitive hostand its prey.
Unsporulatedoocysts are secreted in feces. In the non-feline stage, tissue
cystscontaining bradyzoites or oocysts are ingested by the intermediatehost,
including humans.Human infection is acquired by eating raw or undercookedmeat
infected with tissue cysts or by contact with oocysts fromcat feces in contaminated
litter, soil, or water. Prior infection isconfirmed by serological testing, and its
prevalence depends ongeographic locale and parasite genotype.

Most acute maternal infections are subclinical and are detected only by prenatal or
newbornserological screening. In some cases, maternal symptoms mayinclude
fatigue, fever, headache, muscle pain, and sometimes amaculopapular rash and
posterior cervical lymphadenopathy.The incidence and severity of fetal
toxoplasmosis infectiondepend on gestational age at the time of maternal infection.
Risksfor fetal infection increase with pregnancy duration A metaanalysis estimated
the risk to be 15 percent at 13 weeks,44 percent at 26 weeks, and 71 percent at 36
weeks. Conversely, the severity of fetal infectionis much greater in early pregnancy,
and these fetuses are muchmore likely to have clinical findings of infection.

Pregnant women suspected of having toxoplasmosis shouldbe tested. The parasite


is rarely detected in tissue or body fluids.Anti-toxoplasma IgG develops within 2 to 3
weeks after infection,peaks at 1 to 2 months, and usually persists for life
sometimesin high titers. Although IgM antibodies appear by 10 days afterinfection
and usually become negative within 3 to 4 months,they may remain detectable for
years. Thus, IgM antibodiesshould not be used alone to diagnose acute
toxoplasmosis. IgAand IgE antibodies are also useful in diagnosing acute
infection.Toxoplasma IgG avidity increaseswith time. Thus, if a high-avidity IgG
result is found, infectionin the preceding 3 to 5 months is excluded. Multiple
commercialavidity tests are now available that provide a 100-percent
positivepredictivevalue of high avidity confirming latent infection.

No randomized clinical trials have been performed to assessthe benefit and efficacy
of treatment to decrease the risk forcongenital infection. A systematic review of data
from 1.438treated pregnancies found weak evidence for early treatmentto reduce

Udayana University Faculty of Medicine, DME, 2017 30


Study Guide Infection and Infectious Diseases

congenital toxoplasmosis risks.Treatment has been associated with a reductionin


rates of serious neurological sequelae and neonatal demise.

Prenatal treatment is based on two regimensspiramycinalone or a


pyrimethaminesulfonamide combination withfolinic acid. These two regimens have
also been used consecutively. Little evidence supports the use of a specificregimen.
That said, most experts will use spiramycin in womenwith acute infection early in
pregnancy. Pyrimethaminesulfadiazine with folinic acid is selected for maternal
infectionafter 18 weeks or if fetal infection is suspected.

Udayana University Faculty of Medicine, DME, 2017 31


Study Guide Infection and Infectious Diseases

Parvovirus
Human parvovirus B19 causes erythema infectiosum, or fifthdisease. The B19 virus
is a small, single-stranded DNA virusthat replicates in rapidly proliferating cells such
as erythroblast precursors. This can lead to anemia, which isits primary fetal effect.
Only individuals with the erythrocytegloboside membrane P antigen are susceptible.
In women withsevere hemolytic anemiafor example, sickle-cell disease
parvovirus infection may cause an aplastic crisis.

In 20 to 30 percent of adults, infection is asymptomatic.Fever, headache, and flu-like


symptoms may begin in the lastfew days of the viremic phase. Several days later, a
bright redrash with erythroderma affects the face and gives a

Udayana University Faculty of Medicine, DME, 2017 32


Study Guide Infection and Infectious Diseases

slappedcheekappearance. The rash becomes lacelike and spreads tothe trunk and
extremities.Adults often have milder rashesand develop symmetrical polyarthralgia
that may persist severalweeks.

There is vertical transmission to the fetus in up to a third ofmaternal parvovirus


infections Fetal infection has been associated with abortion,nonimmune hydrops,
and stillbirth. In a review of 1089 cases of maternalB19 infection from nine studies,
Crane (2002) reportedan overall fetal loss rate of 10 percent. It was 15 percent
forinfections before 20 weeks but was only 2.3 percent after 20weeks.

Depending on gestational age, fetal transfusion for hydropsmay improve outcome in


some cases. Mortality rates as high as 30percent have been reported in hydropic
fetuses without transfusions.With transfusion, 94 percent of hydrops cases
resolvewithin 6 to 12 weeks, and the overall mortality rate is < 10 percent.Most
fetuses require only one transfusion because hemopoiesisresumes as infection
resolves.

There is currently no approved vaccine for human parvovirusB19, and there is no


evidence that antiviral treatment prevents maternal or fetal infection. Decisionsto
avoid higher-risk work settings are complex and requireassessment of exposure
risks. Pregnant women should becounseled that risks for infection approximate 5
percent forcasual, infrequent contact; 20 percent for intense, prolongedwork
exposure such as for teachers; and 50 percent for close,frequent interaction such as
in the home. Workers at day-carecenters and schools need not avoid infected
children becauseinfectivity is greatest before clinical illness. Finally, infectedchildren
do not require isolation.

RubellaGerman Measles
This RNA togavirus typically causes infections of minorimportance in the absence of
pregnancy. Rubella infection inthe first trimester, however, poses significant risk for
abortion andsevere congenital malformations. Transmission occurs via
nasopharyngealsecretions, and the transmission rate is 80 percentto susceptible
individuals. The peak incidence is late winterand spring.

Udayana University Faculty of Medicine, DME, 2017 33


Study Guide Infection and Infectious Diseases

Maternal rubella infection is usually a mild, febrile illnesswith a generalized


maculopapular rash beginning on the faceand spreading to the trunk and
extremities. Other symptomsmay include arthralgias or arthritis, head and neck
lymphadenopathy,and conjunctivitis. The incubation period is 12to 23 days. Viremia
usually precedes clinical signs by about aweek, and adults are infectious during
viremia and through 5 to7 days of the rash. Up to half of maternal infections are
subclinicaldespite viremia that may cause devastating fetal infection.

Rubella may be isolated from the urine, blood, nasopharynx,and cerebrospinal fluid
for up to 2 weeks after rash onset. Thediagnosis is usually made, however, with
serological analysis.Specific IgM antibody can be detected using enzyme-
linkedimmunoassay from 4 to 5 days after onset of clinical disease,but it can persist
for up to 6 weeks after appearance of therash.Importantly, rubella reinfection
cangive rise to transient low levels of IgM. Serum IgG antibodytiters peak 1 to 2
weeks after rash onset. This rapid antibodyresponse may complicate serodiagnosis
unless samples are initiallycollected within a few days after the onset of the rash.If,
for example, the first specimen was obtained 10 days afterthe rash, detection of IgG
antibodies would fail to differentiatebetween very recent disease and preexisting
immunity torubella. IgG avidity testing is performed concomitant with theserological
tests above. High-avidity IgG antibodies indicatean infection at least 2 months in the
past.

Rubella is one of the most complete teratogens, and sequelaeof fetal infection are
worst during organogenesis. Pregnant women with rubella infection anda rash
during the first 12 weeks of gestation have a fetuswith congenital infection in up to
90 percent of cases
At 13 to 14 weeks gestation, thisincidence was 54 percent, and by the end of the
second trimester,it was 25 percent. Defects are rare after 20 weeks.

There is no specific treatment for rubella. Droplet precautionsfor 7 days after the
onset of the rash are recommended. Primaryprevention relies on comprehensive
vaccination programs.To eradicate rubella and prevent congenital rubella
syndromecompletely, a comprehensive approach is recommendedfor immunizing
the adult population. MMRvaccine should be offered to nonpregnant women of
childbearingage who do not have evidence of immunity whenever theymake contact
with the health-care system. Vaccination of allsusceptible hospital personnel who
might be exposed to patientswith rubella or who might have contact with pregnant
womenis important. Rubella vaccination should be avoided 1 monthbefore or during
pregnancy because the vaccine contains attenuatedlive virus. Although there is a
small overall theoretical riskof up to 2.6 percent, there is no observed evidence that
the vaccine induces malformations. MMR vaccination is not an indication for
pregnancy termination.Prenatal serological screening for rubella is indicated for
allpregnant women. Women found to be nonimmune should beoffered the MMR
vaccine postpartum.

Udayana University Faculty of Medicine, DME, 2017 34


Study Guide Infection and Infectious Diseases

Cytomegalovirus
This ubiquitous DNA herpes virus eventually infects mosthumans. Cytomegalovirus
(CMV) is the most common perinatalinfection in the developed world. Specifically,
someevidence of fetal infection is found in 0.2 to 2.5 percent ofall neonates. The
virus is secreted into allbody fluids, and person-to-person contact with viral-
ladensaliva, semen, urine, blood, and nasopharyngeal and cervicalsecretions can
transmit infection. The fetus may becomeinfected by transplacental viremia, or the
neonate is infectedat delivery or during breast feeding.

Primary maternal CMV infection is transmitted to the fetusin approximately 40


percent of cases and can cause severe morbidity. In contrast, recurrentmaternal
infection infects the fetus in only 0.15 to 1 percentof cases. A review of nine studies
of CMV vertical transmissionrates reported first-trimester transmission in 36 percent,
second-trimester in 40 percent, and third-trimester in 65 percent. Naturally acquired
immunity during pregnancyresults in a 70-percent risk reduction of congenital CMV
infection in future pregnancies. However, asnoted earlier, maternal immunity does
not prevent recurrences,and maternal antibodies do not prevent fetal infection.
Also,some seropositive women can be reinfected with a differentviral strain that can
cause fetal infection and symptomatic congenitaldisease.
Routine prenatal CMV serological screening is currently notrecommended. Pregnant
women should be tested for CMV ifthey present with a mononucleosis-like illness or
if congenitalinfection is suspected based on abnormal sonographic findings.Primary
infection is diagnosed using CMV-specific IgGtesting of paired acute and
convalescent sera. CMV IgM doesnot accurately reflect timing of seroconversion
because IgMantibody levels may be elevated for more than a year. Moreover, CMV
IgM may be found with reactivationdisease or reinfection with a new strain. Thus,
specificCMV IgG avidity testing is valuable in confirming primaryCMV infection. High
anti-CMV IgG avidity indicatesprimary maternal infection > 6 months before testing.
Finally, viral culture may be useful,although a minimum of 21 days is required before
culturefindings are considered negative.

Udayana University Faculty of Medicine, DME, 2017 35


Study Guide Infection and Infectious Diseases

Several fetal abnormalities associated with CMV infectionmay be seen with


sonography, computed tomography, or magneticresonance imaging. In some cases,
they are found at thetime of routine prenatal sonographic screening, but in
othersthey are part of a specific evaluation in women with CMVinfection. Findings
include microcephaly, ventriculomegaly,and cerebral calcifications; ascites,
hepatomegaly, splenomegaly,and hyperechoic bowel; hydrops; and
oligohydramnios.

The management of the immunocompetent pregnant womanwith primary or


recurrent CMV is limited to symptomatictreatment. If recent primary CMV infection is
confirmed,amnionic fluid analysis should be offered. Counseling regardingfetal
outcome depends on the gestation age during whichprimary infection is
documented. Even with the high infectionrate with primary infection in the first half of
pregnancy, mostfetuses develop normally.

There is no CMV vaccine. Prevention of congenital infectionrelies on avoiding


maternal primary infection, especially inearly pregnancy. Basic measures such as
good hygiene and handwashing have been promoted, particularly for women with
toddlersin day-care settings. Although there maybe sexual transmission from
infected partners, there are no dataon the efficacy of preventive strategies.

Hepatitis B
Chronic hepatitis B virus (HBV) infection is estimated to affect >350 million people
worldwide and represents a significant cause ofmorbidity and mortality related to
cirrhosis and hepatocellular carcinoma. Mother-to-child transmission (MTCT) of HBV
remainsan important source of incident cases of HBV. Current barriers to eradication
of incident HBV infections via MTCT include underutilizationof immunoprophylaxis
with hepatitis B vaccination and hepatitis B immune globulin in certain endemic
regions aswell as failure of immunoprophylaxis.

Hepatitis B perinatal transmission remains a common mode ofviral transmission,


especially in highly endemic areas globally.The availability over the past decade of
effective oral agents thatsuppress viral replication has allowed the consideration of
thirdtrimestertreatment to reduce the risk of this transmission. Thisis important,
particularly in pregnant women with very highviral levels (>108 copies/mL or 2 107

Udayana University Faculty of Medicine, DME, 2017 36


Study Guide Infection and Infectious Diseases

IU/mL), in whom therisk is highest, but transmission can occur even at levels>200
000 IU/mL. Treatment decisions necessitate careful discussionof risks and benefits
as emerging data suggest some possibleeffect on bone mineral concentration in
tenofovir-exposedpregnant women, which must be balanced by a nearly 10% riskof
chronic infection with an incurable virus. Pregnant womenwith HBV must be
monitored for clinical flares, with or withoutmedications, and breastfeeding should be
allowed as well.

In the absence of HBV immunoprophylaxis, 10 to 20 percentof women positive for


HBsAg transmit viral infection totheir infant. This rate increases to almost 90 percent
if themother is HBsAg and HBeAg positive. Immunoprophylaxisand hepatitis B
vaccine given to infants born to HBV-infectedmothers has decreased transmission
dramatically and preventedapproximately 90 percent of infections.

Hepatitis C
Hepatitis C virus (HCV) is a well known cause of chronic liver disease in adults, but
the burden of HCV inpregnant women and children is underappreciated. The leading
route of HCV acquisition in children is verticaltransmission.

Women with chronic HCV infection often have uneventfulpregnancies without


worsening of liver disease or othermaternal or infant adverse effects; some women
mayeven have improvement. For example, in a series of 266pregnant women
infected with HCV, elevated serum alanineaminotransferase (ALT) levels were
detected in 56%of women at the beginning of pregnancy but only 7% during the
third trimester. However, 55% of womenreturned to elevated ALT levels by 6 months
postpartum.Such changes may be dueto the significant changes in the maternal
immune systemduring pregnancy.

Infants born to women infectedwith HCV were more likely to be low birth weight,
smallfor gestational age, and require neonatal intensive care andassisted
ventilation. In the same cohort, women infectedwith HCV had an increased risk for
gestational diabetesbut only when combined with excessive gestational weightgain.

There is currently no licensed vaccine for HCV prevention.The chronic HCV


infection treatment has traditionallyincluded alpha interferon (standard and
pegylated), aloneor in combination with ribavirin. This regimen is contraindicatedin
pregnancy because of the teratogenic potential ofribavirin in animals.

Learning task
1. What is IgG avidity ? Who important this result for the treatment scenario ?
2. Describe about indication, side effect, effective dose, contra indication of
spiramycin ?
3. How to protect pregnant women from Parvovirus infection ?
4. Describe about MMR vaccination ?
5. How to protect pregnant women from Cytomegalovirusinfection ?

Self assement
1. Explain how to manage pregnant women with human immunodeficiency
virus ?
2. Explain how to manage pregnant women with Epstein-Barr virus ?

Udayana University Faculty of Medicine, DME, 2017 37


Study Guide Infection and Infectious Diseases

3. Explain how to manage pregnant women with herpesviruses 6 and 8 ?


4. Explain how to manage pregnant women with varicella ?
5. Explain how to manage pregnant women with syphilis ?
6. Explain how to manage pregnant women with enteroviruses ?

Lecture 7
CENTRALNERVOUS SYSTEM INFECTIONS
MENINGITIS ANDENCEPHALITIS

AIMS:
Describe diagnosis, initial management and referral patients with meningitis and
encephalitis

LEARNING OUTCOME:
1) Defineof meningitis and encephalitis
2) Describe prevalence of meningitis and encephalitis
3) Describe the most common signs and symptoms of meningitis and encephalitis
4) Describe the most common causes of meningitis and encephalitis
5) Describe common complication of Meningitis and Encephalitis
6) Explain the patophysiology of Meningitis and Encephalitis
7) Explain neurologic examination for meningitis and encephalitis
8) Discuss the need for urgent investigations and referrals

CURRICULUM CONTENS:
1. History taking patients with Meningitis and Encephalitis
2. Physical Examination patients with Meningitis and Encephalitis
3. Initial Management patients with Meningitis and Encephalitis

ABSTRACTS
Central nervous system (CNS) infections presenting to the emergency room
include meningitis, encephalitis, brain and spinal epidural abscess, subdural
empyema, and ventriculitis. These conditions often require admission to an intensive
care unit (ICU) and are complications of ICU patients with neurologic injury,
contributing significantly to morbidity and mortality.CNS infections may have acute
and chronic neurologic sequelae, including seizures, hydrocephalus, focal
neurologic deficits, sensorineural hearing loss, cognitive deficits, and personality
change.
Meningitis is an inflammation of the arachnoid membrane, the pia mater, and
the intervening cerebrospinal fluid (CSF). The inflammatory process extends
throughout the subarachnoid space around the brain and spinal cord and involves
the ventricles. Acute-onset fever, generalized headache, vomiting, and stiff neck are
common to many types of meningitis. Bacterial meningitis is a medical emergency
that requires immediate diagnosis and rapid institution of antimicrobial therapy.
Delay in treatment is the most critical factor in determining the morbidity and
mortality of patients with bacterial meningitis.
Encephalitis is defined by the presence of an inflammatory process of the
brain in association with clinical evidence of neurologic dysfunction. Of the
pathogens reported to cause encephalitis, the majority are viruses.The syndrome of

Udayana University Faculty of Medicine, DME, 2017 38


Study Guide Infection and Infectious Diseases

acute encephalitis shares many clinical features with acute meningitis, such that
patients with either syndrome may present with fever, headache, and altered level of
consciousness.Viral encephalitis is a medical emergency. The spectrum of brain
involvement and the prognosis are dependent mainly on the specific pathogen and
the immunological state of the host.
SELF DIRECTING LEARNING
Basic knowledge that must be known:
1. The Meningens and The cerebrospinal fluid and ventricular system
2. The technique performing a lumbar puncture/spinal tap
3. Cerebrospinalfluidfinding in CNS infection
Scenario1
A 28 year-old man presents the emergency room with a severe headache,
fever and confusion and hearing loss, all of which had developed two days before
visiting hospital. He is not known to have any medical illness, and there is no history
of head trauma. On examination , he has a temperature of 38 0C , Blood pressure
110/68 mmHg and pulse 100 beat/min. Neurologic examination revealed meningeal
irritation (neck rigidity and kernig sign) and No. sign of neurologic focal and deep
tendon reflexes were normal.
Learning Task:
1. What is the most likely diagnosis ?
2. What is the best diagnostic next step ?
3. Whatistheappropriatetheraphy ?
4. What is the differential diagnosis of this patient?
Scenario 2
A 18 year old woman referred to our hospital with fever and seizure. Fever has
been initiated 3 days before admission, On the third day, she developed tonic
seizure with upward gaze lasting 5 minutes, after seizure she had problem in
recognizing her parents. After second seizure she was referred to our hospital. On
admission she was febrile and disoriented. Neurologic examination revealed no sign
of meningeal irritation or neurologic focal sign and deep tendon reflexes were
normal.

Learning Task:
1. What is the most likely diagnosis ?
2. What is the best diagnostic next step ?
3. Whatistheappropriatetheraphy ?
4. What is the differential diagnosis of this patient?
Self Assessment
1. Know the clinical presentation of meningitis and encephalitis
2. Learn to develop a diagnostic strategy for the diagnosis of meningitis ,
encephalitis
3. Know strategy for meningitis, encephalitis and cerebral malaria in the
emergency room

Refference
Darrof R, Jankovic J, Mazziotta J, Pomeroy S. Bradleys Neurology in Clinical
Practice. Elsevier inc. 2016.
Jones H, Srinivasan J, Allam G, Baker R. NETTERS NEUROLOGY. Elsevier Inc.
2012.

Udayana University Faculty of Medicine, DME, 2017 39


Study Guide Infection and Infectious Diseases

Ngoerah IGNG.Dasar-Dasar Ilmu Penyakit Saraf. Udayana UniversityPress.


Denpasar. 2017
Scheld W, Whitley R, Marra C. Infectionof The CentralNervous System. Lipincott
William &Wilkins. 2004.

Lecture 8
THYPOID FEVER

Abstract
Typhoid fever is an acute systemic infection which is caused by Salmonella typhi. S
typhi is a gram-negative bacilli , aerobic, and moving with flagellae. S. typhi enters
the human body form contaminated food, and then penetrate to gut mucosal
membranes, continue to gut lymph nodes . S.typhi become reproductive and then
from thoracic ducts enters to the blood stream, going to the reticuloendothelial
system (first bacteremia), occurs in 24-72 hours after S typhis entrance and rarely
diagnosed diagnosed because its asymptomatic. First bacteremia will soon be
ended after this microorganism is not shattered because of fagocytosis, due to
protection from Vi cappsule. In this organ of reticuloendothelial system, the
microorganism continued to be reproductive. This process lasted in 7 until 10 days.
And then the microorganism continue to enter the blood stream and cause the
secondary bacteremia.
Clinical Findings During prodromal stage, there is increasing malaise, headache,
abdominal pain and constipation, while the fever ascends in astepwise. After 7-10
days, the fever reaches a plateau and the patient is much more ill, appearing
exhausted and often prostrated. There may be marked abdominal distention,
especially early, or pea soup diarrhea; marked abdominal distention occurs as
well. During the early prodrome, physical sign are few. Later, typhoid tongue,
splenomegaly, abdominal distention and tenderness, relative bradicardia appear.
Laboratory findings: Typhoid fever is best diagnosed by blood culture, which is
positive in the first week of illness in 80% patients who have no taken antimicrobial.
Culture of bone marrow occasionally are positive when blood cultures are not. The
other laboratory test including: PCR, specific serologic test: widal
Management: Fluids and electrolytes should be monitored and replaced. Oral
nutrition with a soft digestible diet is preferable in the absence of abdominal
distension or ileus. No specific limitations on activity are indicated for patients with
typhoid fever. As with most systemic diseases, rest is helpful, but mobility should be
maintained if tolerable. The patient should be encouraged to stay home from work
until recovery. Several antibiotics including: ampicillin, azithromycin.
Chlorampenicol, third generation cephalosporins, and quinolon are effective for
treatment.

Learning Task:
Case 1
A 22-year-old male, with feeling generally unwell with fever, headache, malaise and
diarrhea. the onset of fever since 7 days ago. His body temperature was 39 degree
celcius, blood pressure 120/80 mmHg, Pulse rate 100 beat per minute.
1. Define and describe others symptoms related to the patients that should be
asked to this patient
2. Describe physical examination to support diagnosis of this patient.

Udayana University Faculty of Medicine, DME, 2017 40


Study Guide Infection and Infectious Diseases

3. What is possibly diagnosis of this patient?


4. Describe differential diagnosis of this case
5. Describe laboratory and other examination to support the diagnosis
6. Describe management of this patient
7. Describe how to explain to this patient about prognosis of patient`s disease

Self assessment:
1. Explain pathogenesis of Typhoid fever
2. Define signs and symptoms of Typhoid fever
3. Define management of Typhoid fever
4. Describe about complication of Typhoid fever

Lecture 9
ACUTE GASTROENTERITIS

Abstract
Acute gastroenteritis is define as the inflammation of the mucus membranes
of the gastrointestinal tract and is characterized by diarrhea and/or vomiting. It
causes significant mortality in developing countries and significant economic burden
to developed countries. Over 1.7 billion global cases of diarrheal disease are
reported annually and are associated with an estimated 2.2 million deaths. Viruses
are responsible for approximately 70% of episodes of acute gastroenteritis and
rotavirus is the one of the best studies of these viruses. In developing countries,
enteric bacteria and parasites are more prevalent than viruses and typically peak
during the summer months. The most common bacterial causes are Salmonella
species, Campylobacter sp, Shigella sp and Yersinia sp. Vibrio cholera remains a
major cause of gastroenteritis especially after a disaster where sanitation is
compromised. Giardia lamblia, Cryptosporidium sp and Entamoeba histolytica are
common protozoal infection. The initial clinical evaluation of the patient should focus
on assessing the severity of the illness and the need for rehydration and identifying
likely causes on the basis of history and clinical findings. Correcting fluid and
electrolyte disturbance take patients who are dehydrated of febrile or have blood or
pus in their stool. Oral rehydration therapy is as effective as intravenous therapy in
treating mild to moderate dehydration in acute gastroenteritis and is strongly
recommended as the first line therapy. Antimicrobial therapy are reliably helpful only
for patients with bloody diarrhea (most likely shigellosis), suspected cholera with
severe dehydration and serious non-intestinal infections (e.g., pneumonia).
Antimicrobial are to be considered the drugs of choice for empirical treatment of
travelers diarrhea and community-acquired secretory diarrhea when the pathogen
is known. Anti-emetics are not routinely recommended in treating acute
gastroenteritis, but often used because vomiting is unpleasant and a distressing
symptom which can increase the likelihood or dehydration, electrolyte imbalance,
pulmonary aspiration and most importantly the need for intravenous hydration or
hospitalization. Prevention including; water, sanitation, hygiene, safe food and
immunization can substantially reduce the incidence and severity of the disease.

Learning Task
Case 1:

Udayana University Faculty of Medicine, DME, 2017 41


Study Guide Infection and Infectious Diseases

A 61-years-old woman with diarrhea 6-8 times per day, for 1 day prior to admission,
consistency watery and sometime semisolid, without blood or mucus. She
complained with sudden onset of vomiting (more than 10 times/day), abdominal
pain, nausea, headache, muscle-ache, fever and chills. She is a foreign traveler.
Based on an initial examination, patient was conscious, dry mouth, Blood pressure
90/60 mmHg, body temperature was 38,5 C, increasing bowel sound and acral
clamy.
1. Find key words related to this case
2. Describe condition related to key words
3. Define organ system that involved in this condition and find probably
cause of the key words
4. Define differential diagnosis and other examinations to support the
diagnosis
5. Describe kinds of laboratory examination to diagnose
6. Define management of this case
7. Define complication and prognosis
8. Define prevention based on individual, family, and community
Self assessment:
1. Describe how do people get acute gastroenteritis
2. Describe symptoms of acute gastroenteritis
3. Describe people at risk for acute gastroenteritis
4. Describe diagnosis of acute gastroenteritis
5. Define management of acute gastroenteritis
6. Define prevention of acute gastroenteritis

Lecture 10
HIV INFECTION

Abstract
Retroviruses are enveloped viruses, with an RNA genome. The name is derived
from the fact that the virus particle contains an RNA-dependent DNA Polymerase
(Reverse transcriptase). This enzyme converts the RNA genome into DNA, which
then integrates into the host chromosomal DNA. The reverse transcriptase is highly
error prone and rapid genetic variation is a feature of this group.
The most common retrovirus in human is HIV (human immunodeficiency virus).
Since 1981, the first cases presented with immune deficiency syndrome were
detected in the United State of America. Several years after that, in 1984 it was
known that the syndrome was caused by HIV infection. HIV targeted cells who carry
CD4 receptor molecule as the main receptor, though other co-receptor is also
needed for HIV to infect the cells, such as CCR5 and CXCR4. Transmissions:
Commonly through sexual transmission, Blood transfusion, Mother to infant and
rarely accidental occupational exposure
Primary HIV infection may present with the clinical picture of a febrile illness
approximately 24 weeks after exposure. The symptoms may include skin rash,
myalgia, fatigue, sore throat, diarrhoea, lymphadenopathy, hepatosplenomegaly
and, rarely, neurological symptoms. While this seroconversion illness may occur in
up to 7080% of individuals, it is often not viewed as serious or related to HIV
infection, due in part to its self-limited course. Primary HIV infection seldom results
in presentation to health care settings.

Udayana University Faculty of Medicine, DME, 2017 42


Study Guide Infection and Infectious Diseases

In line with the course of time of infection, when the immune system becomes
progressively damaged by HIV, reflected by an increased of HIV viral load and a
steady decline of the CD4 lymphocyte count to 200 350 cells/L, the patient may
develop symptoms that are commonly associated with HIV disease. These include
persistent fever, night sweats, significant weight loss, oral thrush, herpes zoster and
chronic diarrhoea. A range of opportunistic infection will occur. Thrombocytopenia
and lymphopenia may be present on blood testing. It should be noted that any organ
system can become involved in opportunistic infections and the clinical presentation
of acquired immunodeficiency syndrome (AIDS) is therefore highly variable. Please
see WHO Classification for disease staging and also CDC classification of HIV
infection based on laboratory examination and the list of diseases referred as AIDS
defining illnesses (ADI). So, infection with HIV will covered a very wide range, from
asymptomatic to severe clinical manifestation of opportunistic infection.
Diagnosis of HIV infection can be made if there are risk factors of HIV infection,
clinical manifestation of disease mentioned above, especially in late stage will
presented with AIDS defining illness and laboratory test positive for HIV-antibody or
any other test confirming HIV infection, such as P24 Ag test and HIV viral load test.
Treatment to HIV/AIDS, include treatment to the specific manifestation of
opportunistic infection, antiretroviral treatment, supportive treatment, symptomatic
drugs, supportive counseling, psychological treatment and counseling to increased
adherence.

Learning task:
A young man aged 35 years-old came to the hospital with difficulty in swallowing,
dry mouth and fever. He also had diarrhea since a couple months ago, loose 7 kg
of his body weight, and feel unwell to work. Because he cannot eat, he become
very week and also had a dry cough which made him mostly have to stay in bed.
1. What syndrome do this young man suffering from?
2. On history taking, what information do you want to know from the patient to
lead you to a possible working diagnosis? Please explain.
3. What is the diagnosis of this patient?
4. How do you manage this patient?

Self assessments:
1. Describe the pathogenesis of immune deficiency in HIV infection.The clinical
manifestation of HIV infection is very wide spectrum, from very mild and
asymptomatic to a severe- life threatening diseases. How can we make
diagnosis of the infection or diseases?

Lecture 11
LEPTOSPIROSIS

Abstract
Leptospirosis is a widespread zoonosis caused by pathogenic leptospira spp genus.
Transmission of disease to new hosts is facilatated by contact with contaminated
urine or water source, because leptospira can penetrate broken skin or muocsal
surfaces of new hosts. Infection of new host may be asymptomatic or may be
causes acute severe disease: the Weils disease ( acute renal failure, haemorrhagic

Udayana University Faculty of Medicine, DME, 2017 43


Study Guide Infection and Infectious Diseases

diastesis and jaundice) or severe pilmonary haemorrhagic syndrome. The severity


of the disease in human depend on leptospira species, the health and immune
status of the patient. The mechanisms by which leptospira cause disease are not
well understood. There are factors that role in pathogenesis including: toxin
production, entry and invasion to the host, immune mechanisms and surface
lipoprotein leptospira. These mechanism cause pathological alteration is
caharacterizes by the development of vasculitis, endothelial damage, and
inflammatory infiltrates composed of monocytic cell, plasma cells, histiocytes, and
neutrophils. These pathological alteration can cause multiple hemorrhages and
deterioration of organ functions are among the most striking clinical manifestation
acute severe leptospirosis
Antibiotic therapy of leptospirosis including: mild leptospirosis is treated with
doxycycline, ampicillin, or amoxicillin. For severe leptospirosis, intravenous penicillin
G has long been the drug of choice, although the third-generation cephalosporins
cefotaxime and ceftriaxone have become widely used. Alternative regimens are
ampicillin, amoxicillin, or erythromycin. Several other antibiotics may be usefulfor
example, macrolides, fluoroquinolones, and carbapenems but clinical experience
with these agents is more limited.
Severe cases of leptospirosis can affect any organ system and can lead to
multiorgan failure. Supportive therapy and careful management of renal, hepatic,
hematologic, and central nervous system complications are important.

Keyword: Leptospirosis, pathogenesis, clinical manifestation

Reference:
1. Speelman P. Leptospirosis. In: Kasper DL., Braunwald E., Fauci AS., Hauser
SL., Longo DL., Jameson JL. eds. Harrisons Principles of Internal Medicine. 16 th
ed. Volume 1. Mc. Grow-Hill companies. 2005. p.988-91
2. Edwards CN. Leptospirosis. In: Cohen J., Powderly W.G., Berkley S.F., Holland
S.M., Opal S.M., Callandra T. Infectious disease. Mosby. London. 2004. p. 1669-
70.

Learning Task:
Case 1
A 21-year-old, previously healthy man, a farmer by profession, was brought to the
Emergency Room by relatives due to high fever, headache, nausea and vomiting,
and constant sleepiness. At admission, he was lethargic and clinically dehydrated.
At physical examination, conjunctival congestion and petechia in the soft palate
region were observed. Pupils were symmetric and reactive. Sclera: icteric. Axillary
temperature was 38.8C, blood pressure was 130/90mmHg and heart rate was 100
beats/min.
1. Define and describe others symptoms related to the patients that
should be asked to this patient
2. Describe physical examination to support diagnosis of this patient.
3. What is possibly diagnosis of this patient?
4. Describe differential diagnosis of this case
5. Describe laboratory and other examination to support the diagnosis
6. Describe management of this patient
7. Describe how to explain to this patient about prognosis of patient`s
disease

Udayana University Faculty of Medicine, DME, 2017 44


Study Guide Infection and Infectious Diseases

Self assessment:
1. Explain pathogenesis of:leptospirosis
2. Define clinical spectrum of leptospirosis
3. Define signs and symptoms of leptospirosis
4. Define management of leptospirosis
5. Describe about complication of leptospirosis

Udayana University Faculty of Medicine, DME, 2017 45


Study Guide Infection and Infectious Diseases

Lecture 12
FILARIASIS

ABSTRACT
Filariasis is a disease group affecting human and animal, caused by filariae, a
nematode (round worm) parasite from Filariidae order. The transmission of filariasis
from infected individual to uninfected individual via mosquito bite. Mosquitoes of the
genera Aedes, Anopheles, Culex, or Mansonia are the intermediate hosts and
vectors of all species that cause lymphatic filariasis. There were hundreds filarial
species and fortunately only eight use humans as their definitive host. In Indonesia,
there were 241 districs from 26 provinces with endemic filariasis; with national
eradication program, the government targeted by the year 2020, Indonesia will be
free from elephantiasis. The WHO has identified lymphatic filariasis as the second
leading cause of permanent and long term disabilities after leprosy. In lymphatic
filariasis, repeated episode of lymphedema and inflammation lead to lymphatic
damage, chronic edema and elephantiasis of the leg, arms, scrotum, vulva and
breast.
Based on the predilection of filarial infestation within human body, filariasis
divided into three groups: (1) Lymphatic filariasis, caused by Wucheria brancrofti,
Brugia malayi and Brugia timori. These worms occupy the lymphatic system
including lymph nodes, in chronic case may lead to syndrome of elephantiasis. (2)
Subcutaneous filariasis, caused by Loa loa (eye worm), Mansonella streptocerca,
Onchocerca volvulus. These worms occupy subcutaneous layer of the skin at the fat
layer. (3) Serous cavity filariasis caused by Mansonella perstans, Mansonella
ozzardi. They occupy the serous cavity of abdomen.
Diagnosis of filariasis by detection of microfilaria through examination of
blood (microfilariae of all species that can cause lymphatic filariasis, Loaiasis (Loa
loa), Mansonella ozzardi, Mansonella perstans), urine (chyluria in lymphatic
filariasis, in concentrated urine evaluation may revealed microfilaria), skin
( Onchocerca volvulus and M streptocerca infections are diagnosed when
microfilariae are detected in multiple skin-snip specimens from different sites located
on both sides of the body), eye (microfilariae of O volvulus may be detected in the
cornea or anterior chamber of the eye using slit-lamp examination). Imaging study
and histology evaluation may be used to demonstrate and monitor lymphatic
obstruction.
Management of filariasis including antihelmintics (Diethylcarbamazine,
ivermectin, suramin, mebendazole, albendazole, flubendazole), surgerical excision
can be considered in lymphatic filariasis with large hydrocele and scrotal
elephantiasis.

LEARNING TASKS
Case 1
A 32-year-old Indian man presented with tender swelling of the right scrotum after a
fall. He was a migrant worker from India who had come to Bali to work as a yoga
teacher. Although there was history of a fall, he denied injury to the groin. Clinical
examination revealed enlarged lymphnodes inguinal, tender and swollen right
scrotum. Eosinophylia was found on complete blood count. Ultrasound of the
scrotum was performed and revealed multiple oval cystic structures with multiple
internal septa in the paratesticular region, adjacent to the body and tail of the

Udayana University Faculty of Medicine, DME, 2017 46


Study Guide Infection and Infectious Diseases

epididymis. Within one of the cysts, numerous short linear echogenic linear
structures were seen moving vigorously. A small right hydrocele was also present.
Question Case 1
1. What is the clinical diagnosis of this case?
2. What about the management of the patient? (including planning for
diagnostic, treatment, monitoring and education)
3. Describe the prevention strategy that available for this case!

Case 2
A 42 year old female patient came to outpatient clinic due recurrent fever which
lasted 3 -5 days and recurrent for the last 3 weeks. She started to feel painful
swelling from her left axillae since the other day. From physical examination
revealed lymphadenitis axillae with redness, warm and tender on palpation in
surrounding area.
Question case 2
1. What is the clinical diagnosis of this case?
2. What evaluation needed for diagnostic work up?
3. How do you manage this case?
4. How about the prognosis?

SELF ASSESSMENT
1. Define the filarial life cycle!
2. Describe the process on obtaining the microfilaria identification from blood
sample!
3. Differentiate between acute and chronic stage of filariasis (comparison table
including duration of infection, clinical manifestation, diagnostic approach,
treatment and prognosis)
4. Differentiate between mass treatment and selective treatment for filariasis
(indication, duration, type of medication)

REFERENCES
1. Knopp S, Steinmann P, Hatz C, Keiser J, Utzinger J. Nematode infections:
filariases. Infect Dis Clin North Am. 2012 Jun. 26(2):359-81
2. Kazura J. Guerrant R, Walker DH, Weller PF, eds. Tropical Infectious
Diseases: Principles, Pathogens and Practice. Philadelphia, PA: Churchill
Livingstone; 1999. Vol 2: 852.
3. Strengthening the assessment of lymphatic filariasis transmission and
documenting the achievement of elimination. WHO Geneva 2014
4. Lymphatic filariasis: A handbook of practical entomology for national
lymphatic filariasis elimination programmes. WHO Department of control of
neglected tropical diseases 2013

Lecture 13
CHILD IMMUNIZATION
ABSTRACT
Immunization or vaccination is the most effective health program and efficient
than other health program in the prevention of an infectious disease. Immunity of the
body against the disease is the main purpose of the vaccination.
Immunity can be given passive and active. Vaccination and immunization are
the example of artificial active immunity in which the vaccine stimulate the body's

Udayana University Faculty of Medicine, DME, 2017 47


Study Guide Infection and Infectious Diseases

active to make specific antibodies. The vaccine components consist of viruses and
bacteria, both viruses and bacteria attenuated or components of viruses and
bacteria.
Some examples of vaccine are BCG, DPT, Hepatitis B, Hib, measles,
pneumococcus, and others. Immunization is based on the immunization schedule in
accordance with the age.
Learning Task :
1. Please describe about the natural and artificial immunity?
2. Please describe about immunization and vaccination?
3. Please describe about the immunological aspects of the immunization
process?
4. Please describe about the aim of immunization?
5. Please describe about Universal Child Immunization program?
6. Please describe about type of immunizations?
7. Please describe about immunization schedule?
8. Please describe about adverse events following immunization?

Self Assessment
A 2 months old boy came to the clinic to get immunization. The history of his
immunization were BCG, Hepatitis B and Polio. There is no fever, cough, and runny
nose.
1. What immunization should be obtained by the child at this time?
2. What is the aim of giving each of these immunizations?
3. Please describe about how to give DPT immunization, vaccine doses, and
the adverse events following immunization?
4. Please describe about how to give Polio immunization, vaccine doses, and
the adverse events following immunization?

Lecture 14
BACTEREMIA, SEPSIS

Abstract
Sepsis is defined as a known or suspected infection plus systemic
manifestations of infections (eg, traditional systemic inflammatory response
syndrome criteriatachy-cardia, tachypnea, white blood count changes, and
fever/hypothermia as well as other metabolic perturbations or organ
dysfunctions). Severe sepsis is defined as sepsis plus infection-induced organ
dysfunction or infection-induced acute tissue hypoperfusion. Bacteriemia is the
presence of viable bacteria in the blood. Early diagnosis, early antibiotic
administration, and adequate fluid resuscitation are key in decreasing sepsis
morbidity and mortality.

Learning task
1. Explain definition of bacteriemia, sepsis, severe sepsis and septic shock
2. How to manage sepsis, severe sepsis and septic shock

Self assessment
Male, 69 years old was admitted in intensive care unit with severe
pneumonia, impending respiratory failure, blood pressure 80/60, pulse 124 irreguler
per minute and respiratory 29 perminute, temperature 35,2 o Celcius. Physical

Udayana University Faculty of Medicine, DME, 2017 48


Study Guide Infection and Infectious Diseases

examination: rales on paracardial of the right lung. Urine was 25 cc for 4 hours.
Laboratory exam: WBC 23 and metabolic acidosis on blood gas analysis
- What is the diagnosis of this case?
- How to manage this case?

Lecture15
URINARY TRACT INFECTION

Abstract
Urinary tract infection (UTI) refers to a symptomatic bacterial infection
within the urinary tract. This includes a lower urinary tract infection cystitis
(symptomatic infection of the bladder), or an upper urinary tract infection acute
pyelonephritis (symptomatic infection of the kidney). These definitions are based
upon a grouping of symptoms. However, the bacterial infection may extend
beyond the anatomical area suggested by the terminology. Asymptomatic
bacteriuria is present if a patient has two consecutive urine cultures showing >100
000 cfu/mL urine, but does not have any symptoms of a UTI. This is only treated
in certain cases, such as prior to a urological operation, in pregnant women, or in
immunocompromised patients. UTIs may be considered complicated if symptoms
of pyelonephritis emerge, or if a UTI is found in certain patient populations,
including the immunosuppressed, men, pregnant women, diabetics, those with a
history of pyelonephritis, or those with structural abnormalities of the urinary tract.
Etiology of UTI is various, most commonly due to infection with Escherichia
coli species (80-90% of cases). Other causes include Klebsiella, Enterococcus,
Proteus mirabilis and Staphylococcus saprophyticus. Periurethral colonization by
the invading pathogen appears to be the initiating step in a cascade of events
leading to a UTI. Most of the causative organisms are naturally present in the GI
tract, which acts as a natural reservoir for potential UTIs.
Symptoms of UTI are (1) Dysuria, which are due to acute inflammation of
the bladder, resulting in discomfort upon contraction during voiding, (2) Frequency
and urgency : reduced bladder capacity due to inflammatory edema causing
decreased compliance and pain due to bladder distension, (3) Hematuria :
irritated, edematous urinary tract bleeding with voiding, (4) Suprapubic tenderness
which are due to palpation and compression of an inflamed, edematous bladder,
(5) Chills and sweats are inflammatory cascade resulting in a febrile response, (6)
Flank pain (may radiate to groin, often dull and constant), and costovertebral
angle (CVA) tenderness are caused by sudden renal edema, resulting in
increased pressure and capsular distension.
While UTIs may be classified in the literature according to location and symptoms,
it is clinically very difficult to determine the extent of infection based on symptoms
Elderly women may present with only urinary incontinence and no other
symptoms. Urine dipstick test showing positive nitrites or leukocyte esterase are
suggestive of UTI, raising the pretest probability by 25%. Nitrite: positive due to
bacterial reduction of endogenous nitrates to nitrites; classically positive in Gram-
negative Enterobacteriaceae family of enteric uropathogens. However, nitrite
dipstick may also be clinically useful in detecting Enterococcus and
Staphylococcus bacteria. Leukocyte esterase: positive as neutrophil granules
contain enzymes with esterase activity; presence ofneutrophils in urine due to

Udayana University Faculty of Medicine, DME, 2017 49


Study Guide Infection and Infectious Diseases

inflammation andleukocyte migration into the urinary tract. Only a positive urine
culture is considered truly diagnostic of a UTI, however, urine should only be
cultured in the setting ofclinical infection or infective symptoms; as previously
mentioned, asymptomatic bacteriuria is common and does not require treatment.
Treatment with antimicrobials aims to eradicate the bacteria causing
infection. The chosen antimicrobials depend on extent of infection (uncomplicated
or complicated), common local pathogens, and resistance patterns. Examples of
antibiotics for uncomplicated UTI include: (1) Trimethoprim-sulfamethoxazole :
Inhibition of microbial DNA synthesis by inhibiting the folic acid synthesis and
consequently the purines required for DNA, (2) Fluoroquinolones: Inhibition of
microbial DNA synthesis by blocking DNA gyrase and topoisomerase IV needed for
successful DNA replication and transcription, (3) Nitrofurantoin: The mechanism is
not fully understood,but it directly causes selective damage to microbial DNA,which
metabolises the toxic intermediates of nitrofurantoin more rapidly than human cells.

Case:
Lisa, age 14, is brought in to the surgery by her mother with a 2-day history of
fever and irritability. Her mother has noticed that she has been going to the toilet
more often and complaining of pain when she passes urine. Lisa has not been
eating well but has had no episodes of vomiting. She has had no previous urinary
frequency or dysuria that her mother can remember and has no relevant medical
history. She is not taking any medications and has no known allergies. On
examination her pulse rate is 100 beats per minute and temperature is 37.8 C. She
weighs 37 kg and is 142 cm. On palpation her abdomen is soft and her bladder is
not palpable. She has no suprapubic tenderness and does not complain of flank
pain on examination. here is no evidence of vulvitis. A urinalysis was positive for
leukocytes and nitrites. Midstream urine (MSU) was sent for culture and sensitivity.

Learning task:
1. Find keywords related to this case
2. Describe condition related to keywords!
3. Define organ system that involved in this condition and find probably cause of
the keywords!
4. Define differential diagnosis and other examination to support the diagnosis.
What further investigation/s (if any) would you order?
5. Define management of this case!
6. What would be your initial step in drug management (if any) before the MSU
result is available?
7. Define complication and prognosis this case!
8. Define prevention based on individual, family and community!

Self Assessment:
1. Describe how do people get UTI!
2. Describe symptom of UTI?
3. Describe people at risk for UTI
4. Describe pathogenesis of UTI
5. Describe diagnosis of UTI
6. Define management of UTI
7. Define prevention UTI

Udayana University Faculty of Medicine, DME, 2017 50


Study Guide Infection and Infectious Diseases

Lecture 16
LOWER RESPIRATORY TRACT INFECTION

Abstract
Lower respiratory tract infections are less common but are more likely to cause
serious illness and death. Diagnosis and specific chemotherapy of respiratory tract
infections present a particular challenge to both the clinician and the laboratory staff.
Successful preventive strategies are available for several respiratory infections.
Lower respiratory tract infections (LRTIs) is infection below the level of the larynx
and may be taken to include: Bronchiolitis, Bronchitis, Pneumonia,
Laryngotracheobronchitis (croup), Pulmonary tuberculosis. Usually with cough as
the main symptom, with at least one other lower respiratory tract symptom (sputum
production, dyspnoea, wheeze or chest discomfort/pain) and no alternative
explanation (e.g. sinusitis or asthma). The presentation of these conditions will
depend on age, infecting organism and site of infection.
The lower respiratory tract consists of: the trachea, the bronchi, bronchioles, and the
alveoli.
The most common lower respiratory tract infections (LTRIs) include: bronchitis,
pneumonia and pulmonary tuberculosis.
In this lesson were discuss only acute bronchitis and pneumonia. The main
symptom of a lower respiratory tract infection is a cough, although it is usually more
severe and more productive (bringing up phlegm and mucus). Sometimes the
mucus is blood-stained. Other symptoms of a lower respiratory tract infection
include: a tight feeling in your chest, breathlessness, wheezing, sore throat, fever
and chills, headaches, blocked nose and sinuses, aches and pains.
The causatives lower respiratory tract infections are by viruses, bacteria, and rarely
fungi.
History, physical examination, X-rays and laboratory investigations focus on two
issues: the degree of respiratory compromise and the identity of the causal
pathogen. Diagnostic specimens can be obtained from the respiratory tract with
deceptive ease, but their value is often limited by contamination by the indigenous
flora of the oral cavity. A history of tobacco consumption, recent travel, occupation,
pets, and contacts with similar symptoms should be sought.
Antibiotic treatment should be prescribed in patients with suspected or definite
pneumonia. Antibiotic treatment should be considered for patients with LRTI and
serious co-morbidity.
Some patients recommendation to vaccination with influenza vaccine and
pneumococcal.

Case 1:
A 45-years-old man with dyspnea since 5 days prior to admission. Dyspnea
accompanied by productive coughing and high grade fever. He sought treatment
from local district hospital doctor, in 2 days admission, such as; paracetamol 1 gram
intra vena every 8 hours, amoxicillin 1 gram intra vena every 8 hours, and
ambroxole syrup 1 spoon every 8 hours. The treatment given does not works and
then he was referred to Sanglah Hospital for intensive treatment, with diagnosis:
suspected pneumonia cause by bacterial. He is a smoker and some time drinks
beer. Based on an initial examination, patient fully alert, look pale and severe ill,

Udayana University Faculty of Medicine, DME, 2017 51


Study Guide Infection and Infectious Diseases

blood pressure 120 over 80 mmHg, body temperature was 38,8 C, and respiratory
rate 32 times per minute. Chest x-ray with shadowing patchy infiltrate in both side of
lungs.
Learning task:
1. Find key words related to this case
2. Describe condition related to key words
3. Define organ system that involved in this condition and find probably
cause of the key words
4. Define differential diagnosis and other examinations to support the
diagnosis
5. Describe kinds of laboratory or radiology examination to diagnose
6. Define management of this case
7. Define complication and prognosis
8. Define prevention based on individual, family, and community
Self assessment:
1. Describe how do people get pneumonia
2. Describe symptoms of pneumonia
3. Describe people at risk for pneumonia
4. Describe pathogenesis of lobar pneumonia
5. Describe diagnosis of pneumonia
6. Define management of pneumonia
7. Define prevention of pneumonia

Lecture 17
AVIAN INFLUENZA, SARS

Abstract
Bird flu (avian influenza, AI) is an infection caused by the influenza A virus
subtype H5N1, which generally attack birds (birds and chickens). Type A influenza
viruses are members of the Orthomyxoviridae family. On the surface of the virus
type A, there are two glycoproteins: hemagglutinin (H) and neuraminidase (N).
Subtypes based on the nature of H (H1 to H16) and N (N1 to N9). The avian
influenza virus can survive in water up to 4 days at a temperature of 22 0C and more
than 30 days at a temperature of 0 0C. in poultry feces and body sick poultry, the
influenza virus can live long, but died on heating 60 0 C for 30 min, 56 0C for 3 hours
and heating to 80 0 C for 1 minute. The virus will die with detergents, disinfectants
for example formalin, a liquid containing iodine or 70% alcohol.
Mode of transmission:
Transmission of the disease to humans can be through:
- Animals: direct contact with sick poultry or poultry products were sick.
- Environment: air or equipment contaminated with the virus both derived from
poultry feces or secretions are attacked AI
- Human: very limited and inefficient (found some cases in a group / cluster)
Period of communicability:
Period of transmission to humans is 1 day before, until 3-5 days after onset of
symptoms. In children, the infectious period can be up to 21 days.
High risk groups:
Group to watch and at high risk of AI infection are:
- farm workers / poultry processing (including veterinary / animal husbandry
engineer)

Udayana University Faculty of Medicine, DME, 2017 52


Study Guide Infection and Infectious Diseases

- Laboratory workers who process the blood sample / patient secretions /


poultry infected
- The visitors livestock / poultry processing in one last week
- had contact with poultry (chickens, ducks, birds) sick or died suddenly of
unknown causes and or pork as well as raw products in the past 7 days
- AI patients had contact with confirmation in the last 7 days.
Pathogenesis
The pathogenesis is determined by the virulence of the virus and the host immune
response. Factors affecting virulence include high haemaglutinin solution which is
activated by a variety of cellular protease which is a specific substitution in the
polymerase basic protein 2 (Glu627lys) which increasing replication. And a sbstitusi
in nonstructural protein 1 (Asp92Glu) that confers increased resistance to
constraints by interferon and TNF alpha in vitro and extend replication in swine,
increasing release of cytokines, especially TNF alpha in macrophag human
exposure to the virus.
Clinical spectrum
The incubation period: 3 days (average 1-7 days)
Clinical symptoms vary, not all existing symptoms, can vary from person to person,
and tends to quickly deteriorate and develop pneumonia or acute respiratory failure.
Clinical symptoms include: high fever (temperature 380C), cough, runny nose,
sore throat, headache, muscle pain, infection of the lining of the eyes, spasms,
diarrhea or gastrointestinal disturbances, fatigue.
Complication: acute respiratory failure
Laboratory:
confirmation test:
- Culture and identification of influenza A virus subtype H5N1
- RT-PCR testing for H5
Serology:
- Test immunofluorescence assay (IFA): antigen (positive) using a monoclonal
antibody influenza A subtype H5N1
- Neutralization test: there is an increase in specific antibody titers of influenza
A subtype H5N1 4 times, in paired serum neutralization test
Radiolography: chest x rays: ARDS
Diagnosis : Case definitions
suspected cases
- someone who has a fever / suhu 380C accompanied by one or more
symptoms: cough, sore throat, colds and shortness of breath, and followed by
one or more of the following circumstances:
- had contact with poultry (chicken, duck, bird) illness or sudden death
of unknown cause and the raw products (meat, eggs, poultry manure,
etc.) in the last 7 days before onset of symptoms above
- live or have ever been in the area of poultry deaths have unusual (in
large quantities in a short time), in the last 7 days before onset of
symptoms above
- had been in contact with patients confirmed AI cases in the last 7 days
before the onset of the above symptoms.
- had contact with the specimen in the last 7 days before onset of
symptoms above (working in the laboratory for AI)
- leukopenia

Udayana University Faculty of Medicine, DME, 2017 53


Study Guide Infection and Infectious Diseases

- found the H5 antibody titers against the examination HI test using


horse erythrocytes or ELISA test for without subtypes of influenza
- chest x rays found pneumonia picture that quickly deteriorate in the
photo series
OR
- The presence of acute respiratory distress syndrome (ARDS) were not
found another cause with one more condition below:
Leukopenia or relative limfositopeni obtained from counts with
or without thrombocytopenia (below normal values)
chest film depicts atypical pneumonia or pulmonary infiltrates on
both sides of the widening in the photo series
The case probabel
criteria for suspected cases coupled with one or more of the following
circumstances:
- found wearing titer antibodies against H5, a minimum of four times, with the
examination of HI test using horse erythrocytes or ELISA test
- Limited laboratory results positive for influenza H5 (H5 userspecific detection
of antibodies in a single serum) using test neutralisasi (sent to a reference
laboratory)
The cases confirm
probabel suspected cases or with one or more of the following circumstances:
a. AI virus culture / H5N1 Positive
b. PCR AI / H5N1 Positive
c. In asssy immunofluorescence test (IFA) antigen (positive) by using monoclonal
antibodies AI / H5N1
d. The increase in antibody titer AI / H5N1 Positive konvalescen phase (paired sera)
with the neutralization test 4 times the value of the initial (acute phase)
Differential diagnosis:
- typhoid fever
- dengue fever
- pulmonary tuberculosis
- lung infections caused by bacteria or fungi.
Management:
- supportive therapy: oxygen, fluid therapy, nutrition
- Antiviral given as soon as possible (48 hours).
- Body weight > 40 Kg: Oseltamivir 2 X 75 mg per day for 5 days
References
1. Ministry of Health. Guidelines for Management of Avian Influenza in Health
Care Facilities. Jakarta. 2006

Learning Task:
Male 45 years, previously healthy, broilers collectors work, was escorted to the
emergency room, complaining of fever, cough, and shortness of breath since 3
days.. 1 month ago history of some chickens on the farm, many died. on physical
examination found awareness of apathy, Blood pressure: 90/60, respiratory rate: 40
time per minute, pulse rate: 120 time per minute, axillary temperature: 39 oC. On
thorax examination found: ronchy a both lung.
1. Define and describe others symptoms related to the patients that should be
asked to this patient
2. Describe physical examination to support diagnosis of this patient.

Udayana University Faculty of Medicine, DME, 2017 54


Study Guide Infection and Infectious Diseases

3. What is possibly diagnosis of this patient?


4. Describe differential diagnosis of this case
5. Describe laboratory and other examination to support the diagnosis
6. Describe management of this patient
7. Describe how to explain to family this patient about prognosis of patient`s
disease
8. Describe how to referral of this patient

Self assessment:
1. Explain pathogenesis of: avian influenza
2. Define clinical spectrum of avian influenza
3. Define signs and symptoms of avian influenza
4. Define management of avian influenza
5. Describe about complication of avian influenza
6. Define prognosis of avian influenza

Lecture 18
TETANUS

Abstract
Although tetanus can be prevented by appropriate immunization, accidental
tetanus continues to be frequent in underdeveloped and developing countries. The
mortality rate of accidental tetanus varies among different studies and depends on
multiple factors, including patient age, clinical severity; the type of infectious source
wound; incubation and progression times; concomitant respiratory, hemodynamic,
renal and infective complications; site where the patient is treated and the quality of
the care provided, and approach 100% in the absence of medical treatment.
Tetanus is an acute toxin-mediated disease cause by Clostridium tetani, that
is not transmitted from person to person. Clostridium tetani is a Gram positive,
anaerobic, rod- shaped bacterium, whose can developed a terminal spores, survive
in soil and cause infection by contaminating wounds. The spores are very resistant
to heat and the usual antiseptics. Infection occurs when C. tetani spores are
introduced into acute wounds form trauma, surgeries and injections or cronic skin
lesions and infections. Cases have resulted from wounds that were considered too
trivial to warrant medical attention. The incubation period of tetanus is usually
between 3 and 21 days (median 7 days). Outbreaks of tetanus related to injuries
associated with natural disasters such as earthquakes and tsunami have been
documented . Under favourable anaerobic conditions, this ubiquitous bacillus may
produce tetanospasmin, an extremely potent neurotoxin.. Tetanus toxin, the product
of Clostridium tetani, is the cause of tetanus symptoms. Tetanus is taken up into
terminals of lower motor neuron and transported axonally to the spinal cord and /or
brainstem. Here toxin moves trans-synaptically into inhibitory nerve terminals, where
vesicular release of inhibitory neurotransmitter becomes blocked, leading to
disinhibition of lower motor neurons. Muscle rigidity and spasms ensue, often
manifesting as trismus/lockjaw, dysphagia, opistotonus, or rigidity and spams of
respiratory, laryngeal and abdominal muscles, which may cause respiratory failure.
There is no definition for confirmed tetanus. There is no diagnostic laboratory test
for tetanus; the diagnosis is entirely clinical. Probable case defined an acute illness
with muscle spasms or hypertonia and diagnosis of tetanus by a health care
provider or death, with tetanus listed on the death certificate as the cause of death

Udayana University Faculty of Medicine, DME, 2017 55


Study Guide Infection and Infectious Diseases

or a significant condition contributing to death. Differential diagnoses, strychnine


poisoning is the only condition that truly mimics generalized tetanus. Other condition
that can present with some clinical features common to tetanus include: dental
infections (trismus), malignant hyperthermia (hyperthermia and generalized muscle
spasms), stimulant use (trismus, tremor, seizures, tachycardia, and hyperthermia),
atropine poisoning ( trismus, tremor, tachycardia and hyperthermia), hypocalcemia
(trismus and muscle spasms), phenothiazine reaction (trismus and generalized
muscle spasms), acute abdomen (trismus, abdominal muscle spasms and rigidity)
and meningitis (hyperthermia, trismus and muscle spasms). C tetani is recovered
from wounds is only about 30% of cases and the organism is sometimes isolated
from patients who do not have tetanus. Serological results obtained before TIG is
administered can support susceptibility if they demonstrate very low or undetectable
anti-tetanus antibody levels. Acute treatment of tetanus is based on wound cleaning
and antibiotic eradication of Clostridium tetani, e.g. with intravenous metronidazole
500 mg three times daily or penicillin 100.000-200.000 IU/kg/day, treatment is
continued for seven to ten days and the administration of tetanus toxoid and/or
tetanus immunoglobulin. Supportive care and pharmacotherapy are used to control
spasms and manage pain.

Case
A 55 years old man, with chief complain stiffness on the neck and difficulty in
swallowing since 2 days prior to admission, and became worse this evening. He
also had wound on the lower extremities and one puncture deep wound on his left
plantar pedis with pus and full pain, cause by traffic accident 10 days ago. Physical
examination finding Blood pressure 130/80, Pulse Rate 103 beats/mt , Respiratory
rate 22x/mt, body temperature 38.1C, trismus 3 cm and rigidity of abdominal
muscles. Laboratory test revealed a white blood cell count of 6.8x10 3/l (72%
neutrophyls, 11% lymphocytes and 15% monocytes), a haemoglobin of 10.5 g/dl
and a platelet count of 214x10 3/l. Liver enzymes were within normal range.
Learning task
1. Find key words related to this case
2. Describe condition related to key words
3. Define organ system that involved in this condition and find probably cause of
the key words
4. Define differential diagnosis and other examinations to support the diagnosis
5. Describe kinds of laboratory or radiology examination to diagnose e.q.
serology test, imaging, tissue biopsy etc
6. Define management of this case
7. Define complication and prognosis
8. Define prevention based on individual, family, and community

Self assessment:
1. Describe how do people get tetanus
2. Describe symptoms of tetanus
3. Describe people at risk for tetanus
4. Describe diagnosis of tetanus
5. Define management of tetanus
6. Define prevention of tetanus

Udayana University Faculty of Medicine, DME, 2017 56


Study Guide Infection and Infectious Diseases

Lecture 19
SKIN INFECTION

ABSTRACT
Skin Infection is a very common problem in health care setting. This condition can
caused by various agents, including bacteria, virus or fungal group. Skin infection is
not always caused by pathogenic agent, but also by normal flora of the skin such as
Staphylococcus aureus or Candida sp. Examples of skin infection cause by bacteria
are furuncles and erysipelas whereas infection cause by virus including Varicella,
Herpes Zoster and Herpes Simpleks. Skin infection occurred not merely by the role
of infectious agents, but more likely as an interaction of host, agent and
enviromental role. Agent factors consist of the pathogenic or virulence factors of the
agent itself. Enviromental factor includes the temperature, moisture, climate and
enviromental hygiene and crowd. The most important host factor is the function of
the skin as protection barriers. Others includes immunological status of the patient
and also the presence of other systemic disease, Skin infections caused by different
agent group need to be properly diagnosed and treated with appropiate treatment to
avoid serious complications.

LEARNING TASK
1. A 50 years old women came to emergency room, complaint arose some blister of
her left chest since yesterday. It was painful that make her couldnt sleep well.
Since 1 week ago she felt tired and felt burning sensation of that left chest. She
went to cardiogist, the doctor said she was fine.
a. What other anamnesis neded to ask to this patient
b. What are possible differential diagnosis of this patient
c. What is the laboratory examination need to confirm diagnosis
d. How is the management of the patient
e. What are the complication of this patient

SELF ASSESSMENT
1. Explain what we should ask in the anamnesis
2. Explain symptom, sign and what kind of examination do we need for the patient

Lecture 20
SEXUALLY TRANSMITTED INFECTION

CASE
A woman named Wulandari, aged 30 years, currently being pregnant 6
months. This woman came to the Dermato Venereology Clinic, Sanglah Hospital
with complaints of multiple blisters on the labia majora and minora since two days
ago, some blisters have ruptured and caused erosion, she also complained of
soreness when urinating. Such complaints are recurrent, sometimes without
treatment can heal by itself. This is the second pregnancy, whereas the first
pregnancy miscarried at 2 months of age pregnancy.
Her husband also come for treatment, because complaining has blister are
clustered on the shaft of his penis and most of the blister has burst. As was first

Udayana University Faculty of Medicine, DME, 2017 57


Study Guide Infection and Infectious Diseases

symptoms occur 6 months ago. Previously he had never suffered like this, and this
recurrent for the third time

Learning Task:
1. What is the history that needs to be asked again to miss Wulandari?
2. What type of laboratory tests that need to be done to support the diagnosis of
this couple?
3. What are the possible diagnosis of Miss Wulandari and her husband?
4. How is management of the disease are given to Miss Merry and her husband?
5. Is the baby can be infected from the disease suffered by her mother?
6. What action should be done so that the baby is not infected while still in the
womb or during childbirth?
7. To prevent transmission, whether the couple should use condoms during sexual
intercourse?

Udayana University Faculty of Medicine, DME, 2017 58


Study Guide Infection and Infectious Diseases

BASIC CLINICAL SKILL

Udayana University Faculty of Medicine, DME, 2017 59


Study Guide Infection and Infectious Diseases

~ CURRICULUM MAP ~

Program or curriculum blocks


1 Senior Clerkship
0
9 Senior Clerkship
8 Senior Clerkship
Health System- Community- Evidence- Special topics : Elective Study Compre 18
7 based Practice based practice based Medical Health Ergonomy & IV (evaluation) Clinic
(3 weeks) Practice Health Orientation
(4 weeks) Environment (2 weeks) (Clerkship)
BCS (1 weeks) (2 weeks) + medical
(2 weeks) ethic
(4 weeks)
The Medical The Urinary The Reproductive Elective Study 19
6 Cardiovascular Emergency System and System and III
System and (3 weeks) Disorders Disorders
Disorders (3 weeks) (3 weeks) (3 weeks)
(3 weeks) BCS (1 weeks) BCS (1 weeks)
BCS (1 weeks) BCS (1 weeks)
Neuroscience The Respiratory The skin & Special Topic : Forensic Elective 18
5 and System and hearing - Palliative med Medicine and Study II
neurological Disorders system - Complemnt & Medicolegal (2 weeks)
disorders (3 weeks) & disorders Alternative Med. (2 weeks)
(3 weeks) (3 weeks)
BCS (1 weeks) (2 weeks)
BCS (1 weeks) BCS (1 weeks)
Musculoskeletal Alimentary The Endocrine Clinical Nutrition The Visual 18
4 system & & hepatobiliary System, and Disorders system &
connective systems & Metabolism (2 weeks) disorders
tissue disorders disorders and Disorders (2 weeks)
(3 weeks) (3 Weeks) (3 weeks)
BCS (1 weeks) BCS (1 weeks) BCS (1 weeks) BCS (1 weeks) BCS (1weeks)
Behavior Basic Infection Immune Hematologic Special Topic 19
3 Change & infectious system & system & disorder - Andro & aging
and disorders diseases disorders & clinical oncology - -
(3 weeks) (3 weeks) (2 weeks) (3 weeks) Geriat
BCS (1 weeks) BCS (1 weeks) BCS (1 weeks) ri
BCS (1 weeks) -Travel
medicine
- (4
weeks
)
BIOMEDIK III Growth Medical Medical Basic Elective 17
2 (4 weeks) & communicatio Professionalism Pharmaceutical Study I
development n (2 weeks) medicine & (2 weeks)
(2 weeks) (2 weeks) drug etics
BCS: (1 weeks) BCS (1 weeks) (2 weeks)
BCS (1 weeks)
Studium BIOMEDIK I The cell BIOMEDIK II 19
1 Generale and (8 weeks) as (6 weeks)
Humaniora biochemical
(2 weeks) machinery
(2 weeks)
BCS(1 weeks)
Pendidikan Pancasila & Kewarganegaraan ( 3 weeks )

Udayana University Faculty of Medicine, DME, 2017 60


Study Guide Infection and Infectious Diseases

REFFRENCES
1. Spicer WJ. (200): Clinical Bacteriology, Mycology, and Parasitology, An Illustrated
Colour Text. Churchill Livingstone, 14-19.
2. Clinical Bacteriology, Mycology and Parasitology : An Illustrated Colour Text. W.
John Spicer. Churchill-Livingstone
3. Brooks et al. pathogenesis and Control of Viral Diseases. In: Lange Medical
Microbiology. 23rd ed. McGraw Hill. International Ed. 2004. p. 394 413.(Principles
of Viral Infection)
4. Levinson et al. Lange Medical Microbiology & immunology. Examination & Board
review. 8th ed. McGraw Hill. International Ed. 2004. p. 186 220, 259-269, 244-250.
(Principles of Viral Infection)
5. Roitt. I., Brostoff.J., Male. D. Immunology
6. Durack DT, Whitley RJ, and Scheld WM. Introduction: Approach to the Patient with
Central nervous System Infection. In : Scheld WM, Whitley RJ, Durack DT, (eds).
Infections of The Central Nervous System. Raven Press. New York. 1991 p. 1-4.
7. Victor M and ropper AH. Infections of the Nervous System (Bacterial, Fungal,
spirochetal, Parasitic) and Sarcoid. In: Adams and Victors principles of the
Neurology. 7th ed. McGraw-Hill. New York/Toronto. P. 734-780.
8. Ottesen EA. Filariasis.in Powderly WG. (ed). Infectious Diseases. 2nd ed. P.1607-13.
9. Ringsrud KM, Linne JJ. Urinalysis and Body Fluids A Colortext and Atlas. 1st ed.
Mosby. St. Louis/ Toronto. 1995. p. 95-206.
10. Burtis CA. Tietz Fundamentals of Clinical Chemistry. 4th ed. WB Saunders Company.
Philadelphia/ Tokyo. 1996. p. 558-561.
11. Simmons A. Statland BE. Hematology A combined Theoritical and technical
Approach. 2nd ed. Buuterworth-Heinemann. Boston/ Singapore. 1997. p. 129-142.
12. Stites DP, Terr AI, Parslow TG. Medical Immunology. 9th ed. Prentice-Hall
International. 1997. p. 264-269.
13. Kasper DL, Fauci AS, Longo DL, Braunwald E, et al. Harrisons Principles of Internal
Medicine. 16th ed. Vol 1. McGraw-Hill. New York/ Toronto. 2005. p. 981-1103.
14. Sutton D. Radiology and Imaging for Medical Students. Churchill Livingstone. 7 th ed.
1998.
15. Grainger RG and Allison DJ. Diagnostic Radiology. Churchill Livingstone. 2nd ed.
1993.
16. McAdam AJ and Kumar S. Infectious Diseases in Kumar V, Contran RS and Robbins
SL, Robbins Basic Pathology. P. 344-398.
17. Andrews. Diseases of The Skin. 9thed.
18. Bryceson A. Leprosy. 3rded.
19. King & Nicole. Sexually Transmitted Diseases. 2003
20. Holmes KK, Spiring PF, Mirdh P. Sexually Transmitted Diseases. 3rd ed. McGraw-
Hill. 1999.
21. McMillan A, Young H, Ogilvie MM, Scott GR. Clinical Practice in Sexually
Transmissible Infection. Saunders. 2002.
22. Braunwalds Heart Disease. Subacute bacterial endocarditis.

Udayana University Faculty of Medicine, DME, 2017 61

You might also like